all india prelims test series - 2020 · the central government. was the court language. the hindus...

26
1 AIPTS 2020 (HIS - 16) (E) Answer Key Byju’s Classes: 9873643487 ALL INDIA PRELIMS TEST SERIES - 2020 MODERN INDIA – 2 + CURRENT AFFAIRS OF DECEMBER 2019 ANSWER KEY 1. Ans: (c) Explanation: Statement 1 is correct: It introduced Bicameralism at the central level. The act also said there will be gradual decentralization of authority with loosening the supreme hold of the central government. Statement 2 is correct: It introduced Dyarchy in provinces, under which subjects were classified into two categories reserved and transferred. Dyarchy was a gradual transition from irresponsible to responsible government. Statement 3 is incorrect: Under the Morley-Minto Reforms 1909 Indians were given membership to the Imperial Legislative Council for the first time. Statement 4 is correct: The legislature had virtually no control over the Governor - General and his executive council. No bill of the legislature could be deemed to have been passed unless assented to by the governor general. However, the latter could enact a Bill without the assent of the legislature. 2. Ans: (a) Explanation: Following events preceded and influenced formation of All India Muslim League- Statement 1 is correct: With the partition of Bengal and Swadeshi movement, the social separation of the two communities was further politicised by the Swadeshi leaders freely using Hindu religions symbols and forcing Muslims peasants to observe boycott. They unwittingly allowed the movement to grow into the Hindu- Muslim question; instead of having a secular approach to the political issue. Not all the Muslims were separatists or loyalists at the beginning; but the Swadeshi movement soon put on then the unmistakable stamp of otherness. Statement 2 is correct: Morley's Budget speech, 1906 indicated that representative government was going to be introduced in India. This alarmed Muslim leaders across the board, as they thought that in the new self-governing bodies they would be swayed by the Hindu majority who were now well organised under the Congress. This provided the context for the Simla deputation of 1 October 1906 to the Governor General Lord Minto. Statement 3 is correct: The Hindi Urdu controversy: In the United Provinces (now Uttar Pradesh), petitions to the offices and the courts were submitted only in Urdu. as it was the court language. The Hindus for long demanded a change in this procedure. Finally on 8 April, 1900, the government gave instructions that petitions written in Hindi in the Devanagari script would also be accepted. The Muslims resented this directive and called for protest meetings all over the Province. The Hindus held counter meetings and the controversy continued for months, as the rift between the communities widened. This led to communal polarization, thus it is considered one of the reasons which led to the formation of AIML(All India Muslim League). Statement 4 is incorrect: In April 1915, Sarvadeshak (All India) Hindu Mahasabha was formed as an umbrella organization of regional Hindu Sabha, at the Kumbh Mela in Haridwar whereas AIML was established way before in 1906. Thus, it was not influenced by the formation of Hindu Mahasabha. 3. Ans: (b) Explanation: Between the years 1916 and 1918, the Indian independence movement witnessed the growth and spread of the home rule movement spearheaded by leaders like Bal Gangadhar Tilak and Annie Besant. The aim of the home rule movement was the attainment of home rule or a dominion status for India under the British Empire along the lines of countries like Canada and Australia. This movement was carried out through the two home rule leagues by Bal Gangadhar Tilak and Annie Besant. Statement 1 is incorrect: Tilak‘s Home Rule league was restricted to Maharashtra only. While Besant‘s League covered the whole India including Maharashtra. Statement 2 is correct: The Home Rule movement was joined by both moderate leaders (Motilal Nehru, C.R. Das) as well as extremist leaders who were disillusioned by congress inactivity joined the Home Rule league movement.

Upload: others

Post on 10-Apr-2020

0 views

Category:

Documents


0 download

TRANSCRIPT

Page 1: ALL INDIA PRELIMS TEST SERIES - 2020 · the central government. was the court language. The Hindus for long Statement 2 is correct: It introduced Dyarchy in provinces, under which

1 AIPTS 2020 (HIS - 16) (E) Answer Key Byju’s Classes: 9873643487

ALL INDIA PRELIMS TEST SERIES - 2020 MODERN INDIA – 2 +

CURRENT AFFAIRS OF DECEMBER 2019

ANSWER KEY

1. Ans: (c) Explanation:

Statement 1 is correct: It introduced

Bicameralism at the central level. The act also

said there will be gradual decentralization of

authority with loosening the supreme hold of

the central government. Statement 2 is correct: It introduced

Dyarchy in provinces, under which subjects

were classified into two categories reserved

and transferred. Dyarchy was a gradual

transition from irresponsible to responsible government.

Statement 3 is incorrect: Under the

Morley-Minto Reforms 1909 Indians were

given membership to the Imperial Legislative

Council for the first time.

Statement 4 is correct: The legislature had virtually no control over the Governor -

General and his executive council. No bill of

the legislature could be deemed to have been

passed unless assented to by the governor

general. However, the latter could enact a Bill without the assent of the legislature.

2. Ans: (a)

Explanation: Following events preceded and

influenced formation of All India Muslim League-

Statement 1 is correct: With the partition of

Bengal and Swadeshi movement, the social

separation of the two communities was

further politicised by the Swadeshi leaders

freely using Hindu religions symbols and forcing Muslims peasants to observe boycott.

They unwittingly allowed the movement to

grow into the Hindu- Muslim question;

instead of having a secular approach to the

political issue. Not all the Muslims were separatists or loyalists at the beginning; but

the Swadeshi movement soon put on then the

unmistakable stamp of otherness.

Statement 2 is correct: Morley's Budget

speech, 1906 indicated that representative

government was going to be introduced in India. This alarmed Muslim leaders across

the board, as they thought that in the new

self-governing bodies they would be swayed

by the Hindu majority who were now well

organised under the Congress. This provided the context for the Simla deputation of 1

October 1906 to the Governor General Lord

Minto. Statement 3 is correct: The Hindi Urdu

controversy: In the United Provinces (now

Uttar Pradesh), petitions to the offices and

the courts were submitted only in Urdu. as it

was the court language. The Hindus for long

demanded a change in this procedure. Finally on 8 April, 1900, the government gave

instructions that petitions written in Hindi in

the Devanagari script would also be accepted.

The Muslims resented this directive and

called for protest meetings all over the Province. The Hindus held counter meetings

and the controversy continued for months, as

the rift between the communities widened.

This led to communal polarization, thus it is

considered one of the reasons which led to

the formation of AIML(All India Muslim League).

Statement 4 is incorrect: In April 1915,

Sarvadeshak (All India) Hindu Mahasabha

was formed as an umbrella organization of

regional Hindu Sabha, at the Kumbh Mela in Haridwar whereas AIML was established way

before in 1906. Thus, it was not influenced by

the formation of Hindu Mahasabha.

3. Ans: (b) Explanation: Between the years 1916 and

1918, the Indian independence movement

witnessed the growth and spread of the home

rule movement spearheaded by leaders like

Bal Gangadhar Tilak and Annie Besant. The aim of the home rule movement was the

attainment of home rule or a dominion status

for India under the British Empire along the

lines of countries like Canada and Australia.

This movement was carried out through the

two home rule leagues by Bal Gangadhar Tilak and Annie Besant.

Statement 1 is incorrect: Tilak‘s Home Rule

league was restricted to Maharashtra only.

While Besant‘s League covered the whole

India including Maharashtra.

Statement 2 is correct: The Home Rule movement was joined by both moderate

leaders (Motilal Nehru, C.R. Das) as well as

extremist leaders who were disillusioned by

congress inactivity joined the Home Rule

league movement.

Page 2: ALL INDIA PRELIMS TEST SERIES - 2020 · the central government. was the court language. The Hindus for long Statement 2 is correct: It introduced Dyarchy in provinces, under which

2 AIPTS 2020 (HIS - 16) (E) Answer Key Byju’s Classes: 9873643487

Statement 3 is correct: Through seminars, public meetings, libraries, newspapers, and

pamphlets, the Home rule league movement

promoted political education among youths.

4. Ans: (a) Explanation: Recently, the 126th

Constitutional Amendment Bill was passed

by Parliament.

Statement 1 is correct: It amends article

334 to extend reservation only for Scheduled castes (SC) and Scheduled Tribes (ST) to Lok

Sabha and state legislative assemblies. Article

334 originally provided that reservation of

seats and special representation would cease

10 years after the commencement of the Constitution.

Statement 2 is correct: The bill extends

reservation only for Scheduled castes (SC)

and Scheduled Tribes (ST) to Lok Sabha and

legislative bodies till 25th January, 2030

(which was expiring in 2020). Statement 3 is incorrect: The bill does not

propose an extension of reservation quota for

the Anglo-Indian community, which is also

set to expire on January 25. Also, there is no

provision for nominating Anglo Indians to Rajya Sabha.

5. Ans: (c)

Explanation:

Statement 1 is correct: On Gokhale's advice, Gandhiji spent a year travelling

around British India, getting to know the land

and its peoples. Gopal Krishna Gokhale was

undoubtedly the political guru of Mahatma

Gandhi. The two of them first met in 1896

and the connection grew stronger and stronger by the year. Gandhi often wrote to

Gokhale from South Africa seeking his advice

on matters of politics. Gokhale visited Gandhi

in South Africa in 1912, and raised money for

his work. Gokhale was the one who persuaded Gandhi to come back to India,

invest time in understanding India and

engage in the independence struggle.

Statement 2 is correct: The famous

American journalist Mr. Louis Fischer came

to India in May 1942. He was in India for two months. The world war was in full swing. In

the scorching heat of June he spent one week

with Gandhiji in this Ashram. Gandhiji gave

him one hour daily. Fisher wrote a book

"seven days with the Mahatma ''. He also wrote ―The life of Mahatma Gandhi '' In this

famous book he has described the Ashram

life and Gandhi's likes and dislikes in a very

touching way.

6. Ans: (d) Explanation:

Statement 1 is correct: Portfolio System was

introduced by Lord Canning, under which,

a member of the Viceroy's council was made in-charge of one or more departments of the

government and was authorised to issue final

orders on behalf of the council on matters of

his departments.

Statement 2 is correct: Indian Council Act

of 1909 provided for the first time the association of Indians with the executive

council of the viceroy and governors.

Satyendra Prasad Sinha (S.P. Sinha) became

the first indian to join the executive council of

Viceroy as a Law Member. Statement 3 is correct: Under the Indian

Council Act 1861 the Governor-General also

had the power to promulgate ordinances

without the council‘s concurrence during

emergencies.

7. Ans: (d)

Explanation: The Simon Commission‘s report

was published in 1930. Before the

publication, the government assured that

henceforth, Indian opinion would be considered and that the natural outcome of

constitutional reforms would be dominion

status for India.The Simon Commission led to

the Government of India Act 1935 which

acted as the basis for many parts of the current Indian Constitution.

Statement 1 and 3 are correct: It

recommended the abolition of diarchy and the

setting-up of representative governments in

the provinces.

Statement 2 is correct: It also recommended the retention of separate

communal electorates until the communal

tensions had died down.

Statement 4 is correct: It recommended

that Burma should be separated from British India and provided a constitution of its own.

8. Ans: (a)

Explanation: Exercise Apaharan is an anti-

hijacking campaign conducted by the Indian Navy in collaboration with Indian Coast

Guard, Cochin Port Trust, and other

concerned stakeholders. It was aimed at

streamlining the response mechanism or

preparedness to thwart any attempt by anti-national elements to hijack a merchant‘s

vessel. This was the first time that such a

large-scale exercise involving all stakeholders

was conducted in Kerala. Multiple agencies

participated in the exercise. It included 12

ships and helicopters of Cochin Port trust,

Page 3: ALL INDIA PRELIMS TEST SERIES - 2020 · the central government. was the court language. The Hindus for long Statement 2 is correct: It introduced Dyarchy in provinces, under which

3 AIPTS 2020 (HIS - 16) (E) Answer Key Byju’s Classes: 9873643487

Indian Navy and Indian Coast Guard. Hijacking of merchant vessels is one of the

challenging scenarios of the Indian Navy.

Piracy returned in the 21st century after

Somalian pirates began to hijack commercial

ships and demand large ransoms. The Indian Navy began its anti-piracy operations in the

Gulf of Aden in 2008. It escorts Indian-

flagged ships and also ships from other

countries.

9. Ans: (c) Explanation:

Statement 1 is incorrect: It is a joint

initiative by People Strong, a Global Talent

Assessment Company, in collaboration with Confederation of Indian Industry (CII) along

with partners like UNDP, AICTE, and AIU.

Statement 2 is correct: It also indicated the

rising share of gig workers in the economy at

13% share in the overall hiring intent by

employment type. Statement 3 is incorrect: Female

employability witnessed an upward trend at

47 per cent this year from 38 per cent in

2017 and 46 per cent in 2018. A decline in

employability was seen in BTech, Engineering. MCA graduates, Technical &

Computer-related courses.

Statement 4 is correct: Mumbai and

Hyderabad were the top two employable

cities. Maharashtra, Tamil Nadu and Uttar

Pradesh were the top three states in terms of employability.

10. Ans: (a)

Explanation:

Statement 1 is correct: The Revolt of 1857 served as a jolt to the British government.

There was widespread resentment as the

policies of the company were blamed for the

revolt. So, the Act of 1858 ended the doctrine

of lapse. This act was known as 'act for the good government of India'. It was focussed

upon the improvement of administrative

machinery by which Indian government was

to be supervised and controlled in England. It

wanted to bring administrative reforms in the

system. Statement 2 is incorrect: Indian Council

Act, 1861 started the process of

decentralization by restoring the legislative

powers to the Bombay and Madras

Presidencies, which were taken away by Regulating Act 1773.

Statement 3 is incorrect: Pitt's India Act of

1784 distinguished the commercial and

political functions of the company.

11. Ans: (b) Explanation: The correct chronology is as

follows:

1. Alipore Bomb Conspiracy Case (1908): It

is also called Muraripukur conspiracy or Maniktala bomb conspiracy. Revolutionaries

who threw bomb on the carriage of magistrate

kingsford were Prafulla Chaki and Khudiram

Bose. Chaki committed suicide while Bose,

then only 18 years of age, was caught and

sentenced to death by hanging. The other people who were tried in the case were

Aurobindo Ghosh and his brother Barin

Ghosh, Kanailal Dutt, Satyendranath Bose

and more than 30 others. They were all

members of the Anushilan Samiti in Calcutta. Aurobindo Ghosh was acquitted due to lack

of evidence and others served varying life-

terms in prison.

2. Curzon Wyllie's Assassination (1909):

The India House was an organisation in

London involved in the freedom struggle of India mainly engaging Indian students in the

UK as its participants. Patrons of this

organisation included Shyamji Krishna

Varma and Bhikaiji Cama. India House

became the centre of revolutionary activities for Indian independence outside India. The

organisation was liquidated after the

assassination of an army officer Curzon

Wyllie by its member Madan Lal Dhingra in

1909.

3. Delhi-Lahore Conspiracy Case (1912): It is also known as the Delhi Conspiracy Case.

This was an assassination attempt on Lord

Hardinge, the then Viceroy of India. The

revolutionaries were led by Rashbehari Bose.

A homemade bomb was thrown into the viceroy‘s howdah (elephant-carriage) during a

ceremonial procession in Delhi. The occasion

was the transfer of the British capital from

Calcutta to Delhi. Lord Hardinge was injured

while an Indian attendant was killed. Bose

escaped being caught whereas a few others were convicted for their roles in the

conspiracy.

4. Chittagong Armoury Raid (1930): It is

also known as Chittagong Uprising. This was

an attempt by revolutionaries to raid the police armoury and the auxiliary forces

armoury from Chittagong (now in

Bangladesh). They were led by Surya Sen.

Others involved were Ganesh Ghosh,

Lokenath Bal, Pritilata Waddedar, Kalpana

Dutta, Ambika Chakraborty, Subodh Roy, etc. The raiders were not able to locate any

arms but were able to cut telephone and

telegraph wires. After the raid, Sen hoisted

the Indian flag at the police armoury. Many of

the revolutionaries involved escaped but some

Page 4: ALL INDIA PRELIMS TEST SERIES - 2020 · the central government. was the court language. The Hindus for long Statement 2 is correct: It introduced Dyarchy in provinces, under which

4 AIPTS 2020 (HIS - 16) (E) Answer Key Byju’s Classes: 9873643487

were caught and tried. The government came down heavily on the revolutionaries. Many

were sentenced to imprisonment, deported to

the Andaman, and Surya Sen was sentenced

to death by hanging. Sen was brutally

tortured by the police before he was hanged.

12. Ans: (c)

Explanation: Kalpana Dutt was amongst the

initial members of the armed independence

movement led by prominent Bengali revolutionary, Surya Sen. She was arrested

and brought to trial in what was known as

the Chittagong Armoury Raid Supplementary

Case, in which she was sentenced for life.

After the nationwide campaign for the release of the imprisoned Bengal revolutionaries, she

came out of prison in 1939. During the

Bengal famine, one saw her totally devoted to

organising a relief kitchen for the starving

and medical relief for the sick in the

Chittagong villages. In 1943, about the time of the Communist Party Congress, Kalpana

married P.C. Joshi, the popular leader of the

Communists. She was fully occupied with her

party work in Bengal. When the communal

holocaust of the partition overtook Bengal, Kalpana was equally active in relief and

rescue work. Then came the period of

insensate sectarian adventurism of the Indian

Communists under Ranadive, inflicting

severe loss on the movement. Joshi and

Kalpana were thrown out of the party. Bereft of shelter but undaunted in spirit, Kalpana

received support from close friends, one of

whom was Prof P.C. Mahalanobis who

engaged her in his Statistical Institute.

13. Ans: (c)

Explanation:

Operation Clean Art was the first pan India operation to crackdown on the

smuggling of mongoose hair conceived by

Wildlife Crime Control Bureau.

It aims to ensure that the mongoose hair brush trade should be closed down across

the country.

As per IUCN, Mongoose is a least concerned species. Mongoose is listed in

Schedule II Part 2 of the Wildlife

Protection Act. Any smuggling or

possession of its body part is a non-bailable offence.

14. Ans: (a) Explanation:

Statement 1, 2 and 4 are correct:

Recommendations of the report were-

Dominion status for India (like Canada, Australia, etc.) within the British Commonwealth. (This point was a bone of

contention with the younger set of leaders

including Jawaharlal Nehru and Subhas

Chandra Bose who favoured complete

independence.)

Nineteen fundamental rights including the right to vote for men and women above 21 years of age, unless

disqualified.

Equal rights for men and women as citizens.

No state religion.

No separate electorates for any community. It did provide for reservation

of minority seats. It provided for

reservation for seats for Muslims at the centre and in provinces where they were

in a minority and not in Bengal and

Punjab. Similarly, it provided for

reservation for non-Muslims in the NWFP.

There would be a bicameral legislature at the centre. The ministry would be

responsible to the legislature.

Governor-General to be the constitutional head of India. He would be appointed by

the British monarch.

A proposal for the creation of a Supreme Court.

The provinces would be created along linguistic lines.

The language of the country would be Indian, written either in Devanagari

(Sanskrit/Hindi), Telugu, Tamil, Kannada, Bengali, Marathi or Gujarati in character.

English usage to be permitted.

Statement 3 is incorrect: It proposed

formation of a federal form of government

with residuary powers in the hands of Centre, and not with the provinces.

15. Ans: (a)

Explanation:

Statement 1 is correct: The system of preparing an annual budget and laying it

before the Legislature was first introduced in

India in 1860 by James Wilson who was a

Member of the British Parliament, sent to

India as Finance Member of the Viceroy‘s

Council. Statement 2 is incorrect: The Act of 1892

conceded to both the Central and Provincial

Councils the privilege of financial criticism or

Page 5: ALL INDIA PRELIMS TEST SERIES - 2020 · the central government. was the court language. The Hindus for long Statement 2 is correct: It introduced Dyarchy in provinces, under which

5 AIPTS 2020 (HIS - 16) (E) Answer Key Byju’s Classes: 9873643487

the right to discuss the budget under certain conditions for the first time

Statement 3 is incorrect - The right to ask

questions for seeking information from the

government was provided in 1892 but the

right to ask supplementary questions was conceded in 1909.

16. Ans: (b)

Explanation: The object of the Sergeant plan

was to create in India in a period of not less than forty years, the same standard of

educational attainments as had already been

admitted in England.

Statement 1 is correct: Sargent Plan of

Education (1944) is also known as the National Scheme for Education.

Statement 2 is correct: One of its

recommendations was providing free,

universal and compulsory education for

children between the age group of 6 to 11

years. Statement 3 is incorrect: It recommended

that the medium of instruction in all High

Schools should be the mother- tongue of the

pupils. English should be a compulsory

second language.

17. Ans: (a)

Explanation:

Statement 1 is correct: Rabindranath

Tagore became the first Asian to win the Nobel Prize in literature. The Nobel Prize in

Literature 1913 was awarded to

Rabindranath Tagore "because of his

profoundly sensitive, fresh and beautiful

verse, by which, with consummate skill, he

has made his poetic thought, expressed in his own English words, a part of the literature of

the West."

Statement 2 is incorrect: He was awarded a

knighthood by King George V in 1915. He was

no apologist for colonial rule; after the Jallianwala Bagh massacre 1919, he

relinquished his knighthood to show his

protest. He wrote a letter to Lord Chelmsford

the British viceroy, repudiating his

Knighthood.

18. Ans: (b)

Explanation: “Ocean deoxygenation:

Everyone’s problem” report recently

published by the IUCN highlights that the oceans are increasingly experiencing low

levels of oxygen, which threatens marine

ecosystems. The report represents the largest

peer-reviewed study conducted on oxygen

deoxygenation. It examines the causes,

impacts and potential solutions to ocean

deoxygenation. Climate change and nutrient pollution are the main drivers of

ocean oxygen loss, and ocean oxygen loss is

―closely related to ocean warming and

acidification‖ caused by anthropogenic

carbon dioxide emissions and biogeochemical consequences related to anthropogenic

fertilization of the ocean. The report finds that

ocean regions with low oxygen concentrations

have expanded at all depths of the ocean.

19. Ans: (c) Explanation:

Statement 1 is correct: Led by

Surendranath and Ananda Mohan Bose, the

young nationalists of Bengal founded the Indian Association in July 1876. It organised

an All India National Conference in 1883 and

gave a call for another one in December 1885.

That is why S N Banerjee did not attend the

founding session of the INC because he was

preoccupied with the All India National Conference.

Statement 2 is correct: The INC was

established in 1885, by A.O. Hume, a retired

civil servant who was staying in Shimla post-

retirement. He invited many Indian leaders regarding the Indian cause, and he laid the

foundation of the ‗Indian National Union‘.

But, after the suggestion of Dadabhai Naoroji,

its name was changed to ‗Indian National

Congress‘. The word ‗Congress‘ was taken

from the American Constitution.

20. Ans: (a)

Explanation:

Statement 1 is incorrect: The constitution

for the HRA was drafted by Ram Prasad Bismil in 1924 at Kanpur with the blessing of

Lala Har Dayal. Other prominent members of

the party were Sachindra Nath Sanyal and

Jogesh Chandra Chatterjee (who was also a

member of the Anushilan Samiti). The HRA formed centres at, besides Allahabad, Agra,

Kanpur, Varanasi, Lucknow, Shahjahanpur

and Saharanpur. It also had bomb

manufacturing units at Calcutta and

Deogarh. In 1928, the name of the party was

changed to Hindustan Republican Socialist Association (HSRA) primarily because of

Bhagat Singh‘s insistence under Chandra

Shekher Azad

Statement 2 is correct: The most famous

incident was the Kakori train robbery. This occurred on 9th August 1925. Members of

the party looted a train carrying government

money near Lucknow. An innocent passenger

was killed accidentally in the process. People

involved in the episode were Bismil,

Ashfaqulla Khan, Rajendra Lahiri and Thakur

Page 6: ALL INDIA PRELIMS TEST SERIES - 2020 · the central government. was the court language. The Hindus for long Statement 2 is correct: It introduced Dyarchy in provinces, under which

6 AIPTS 2020 (HIS - 16) (E) Answer Key Byju’s Classes: 9873643487

Roshan Singh. All four were eventually hanged by the government in 1927 for their

involvement. Chandrasekhar Azad was also

involved although he evaded arrest.

Statement 3 is correct: A manifesto for the

party was written by Sanyal titled ‗Revolutionary‘. It contained incendiary

material asking the youth of the country to

join the party and take part in the freedom

struggle. It did not approve of the methods

used by Gandhi and criticised them. The

manifesto stated that it sought to achieve a ‗Federal Republic of the United States of

India‘ after overthrowing British rule.

21. Ans: (a) Explanation:

Statement 1 is correct: The Government of

India Act, 1935 provided for the

establishment of an All-India Federation

consisting of provinces and princely states as

units. Statement 2 is correct: The Act divided the

powers between the Centre and units in

terms of three lists—Federal List (for Centre,

with 59 items), Provincial List (for provinces,

with 54 items) and the Concurrent List (for both, with 36 items). It abolished dyarchy in

the provinces and introduced provincial

autonomy in its place with a Government

responsible to an elected Legislature.

Statement 3 is correct: Dyarchy was

introduced at the Centre. The subjects under the Federal List were divided into two:

Reserved and Transferred. The reserved

subjects were controlled by the Governor-

General who administered them with the help

of three councillors appointed by him. The transferred subjects were administered by the

Governor-General with his Council of

Ministers (not more than 10).

Statement 4 is incorrect: Government of

India Act of 1919 for the first time, separated

provincial budgets from the Central budget.

22. Ans: (b)

Explanation: To find a solution to the

stubble burning issue, India is testing a

Swedish technology — torrefaction that can convert rice stubble into ‗bio-coal‘. It is a

thermal process to convert biomass into a

coal-like material, which has better fuel

characteristics than the original biomass. The

technology involves heating up straw, grass, saw mill residue and wood biomass at 250

degrees Celsius to 350 degrees Celsius.

Torrefied biomass is more brittle, making

grinding easier and less energy intensive.

23. Ans: (c) Explanation:

Statement 1 and 2 are correct: Extremism

or militant nationalism became a

predominant philosophy of Indian nationalism or Indian national movement

from 1905 to 1917. The economic miseries of

the 1890s further exposed the exploitative

character of colonial rule. A feeling started

gaining currency that only the masses were

capable of making the immense sacrifices needed to win freedom. The defeat of the

Italian army by Ethiopians (1896), the Boer

wars (1899-1902) where the British faced

reverses and Japan‘s victory over Russia

(1905) demolished myths of European invincibility. Modern education and rise in

unemployment was one of the causes.

Statement 3 is incorrect: C. R. Das was a

moderate leader. The ideologies of

revolutionaries like Lala Lajpat Rai, Bal

Gangadhar Tilak, Bipin Chandra Pal (the first three called Lal-Bal-Pal leading the extremist

cause in Punjab, Bombay and Bengal

respectively) influenced the masses through

their public speeches and their writings in

magazines and newspaper. Other leaders including Aurobindo Ghosh, Rajnarayan

Bose, A K Dutt, V O C Pillai, and Barindra

Kumar Ghosh also propagated extremist

ideologies.

Statement 4 is correct: The limitations and

the failure of the moderate leaders in getting any significant results from the British

authorities was one the main causes of the

rise of extremism.

24. Ans: (c) Explanation:

Statement 1 is incorrect: The aims of the

National Congress were declared to be

−Promotion of friendly relations among

nationalist political workers residing in different parts of the country; Development

and consolidation of the feeling of national

unity irrespective of caste, religion, or

province; Formulation of popular demands

and their presentation before the

Government; and Training and organization of public opinion in the country. Social

reforms were not their agenda.

Statement 2 is correct: Indian (National)

Social Conference was founded by M.G.

Ranade and Raghunath Rao. It was virtually the social reform cell of the Indian National

Congress. The Conference met annually as a

subsidiary convention of the Indian National

Congress, at the same venue, and focused

attention on social reform. The Conference

advocated intercaste marriages and opposed

Page 7: ALL INDIA PRELIMS TEST SERIES - 2020 · the central government. was the court language. The Hindus for long Statement 2 is correct: It introduced Dyarchy in provinces, under which

7 AIPTS 2020 (HIS - 16) (E) Answer Key Byju’s Classes: 9873643487

kulinism and polygamy. It launched the famous ―Pledge Movement‖ to inspire people

to take an oath to prohibit child marriage.

Statement 3 is correct: In his Young India

published in 1916, the Extremist leader Lala

Lajpat Rai used the safety-valve theory to attack the Moderates in the Congress. Safety

Valve is a theory that Hume formed the

Congress with the idea that it would prove to

be a ‗safety valve‘ for releasing the growing

discontent of the Indians and the Congress

was a product of Lord Dufferin‘s brain. But Modern Indian historians dispute the idea of

a ‗safety valve‘. Historian Bipan Chandra

observes early Congress leaders used Hume

as a catalyst to bring together.

25. Ans: (a)

Explanation: After West Bengal, Kerala

Government too has put on hold all

proceedings for updating the National

Population Register (NPR). Statement 1 is incorrect: NPR is a Register

of usual residents of the country. A usual

resident is defined as a person who has

resided in a local area for the past 6 months

or more or a person who intends to reside in that area for the next 6 months or more. So,

NPR may have foreigners as well.

Statement 2 is correct: Census of India will

cover the entire population in the country

while NPR will also cover all the population

except in the state of Assam. The Union Cabinet, chaired by the Prime Minister, has

approved updation of National Population

Register (NPR) at a cost of Rs. 3,941.35 crore.

Statement 3 is correct: NPR is carried out

under the provisions of the Citizenship Act 1955 and the Citizenship (Registration of

Citizens and issue of National Identity Cards)

Rules, 2003. It was prepared in 2010 and was

subsequently updated in 2015 by seeding it

with Aadhaar.

26. Ans: (c)

Explanation:

Option (a) is incorrect: Confederation of

Indian Industry (CII), trade association

representing the interests of Indian businesses in various sectors, chiefly

including engineering, manufacturing,

consulting, and services. The organization

was founded as the Engineering and Iron

Trades Association (EITA) in 1895. EITA was set up at the end of 1895 with the aim of

pressurising the colonial government to place

government orders for iron and steel and

engineering goods with companies based in

India.

Option (b) is incorrect: Bengal National Chamber of Commerce & Industry, the oldest

indigenous Chamber in India was established

on 2nd February, 1887. Its history is closely

interwoven with India‘s struggle for

independence. It is a pioneer in championing the cause of Indian industries in pre-

independent days. It was formed not on the

advice of Gandhi ji, but by elite members of

Bengali zamindars.

Option (c) is correct: The Federation of

Indian Chambers of Commerce and Industry (FICCI) is an association of business

organizations in India. It was established in

1927, on the advice of Mahatma Gandhi by

GD Birla and Purushottam Das Thakurdas.

In 1930, the FICCI advised its members to boycott the Round Table Conference (RTC)

stating that ‗no conference convened for the

purpose of discussing the problem of Indian

constitutional advance can come to a solution

unless such a conference is attended by

Mahatma Gandhi, as a free man, or has at least his approval.

Option (d) is incorrect: Federation of Indian

Mineral Industries (FIMI) established in the

year 1966 is an all-India apex body to

promote the interests of all mining (including coal), mineral processing, metal making and

other mineral-based industries and to attend

to the problems faced by them.

27. Ans: (a) Explanation: The Karachi Resolution was

passed by the Indian National Congress at its

1931 Karachi session.

Statement 1 and 4 are correct: The

resolution guaranteed basic civil rights of free speech, free press, free assembly and freedom

of association, equality before law irrespective

of caste, creed or sex; neutrality of state in

regard to all religions, elections on basis of

universal adult franchise, and free and

compulsory education. It promised substantial reduction in rent and revenue,

exemption from rent in case of uneconomic

holdings, and relief of agricultural

indebtedness, control of usury, and better

conditions for workers including a living wage, limited hours of work and protection of

women workers. The state shall own and

control key industries and services, mineral

resources, railways, waterways, shipping aid

and other means of public transport.

Statement 2 is incorrect: Total prohibition (not just regulation) of intoxicating drinks

and drugs was proposed.

Statement 3 is correct: The resolution

demanded a living wage (not Minimum wage)

for industrial workers, limited hours of

Page 8: ALL INDIA PRELIMS TEST SERIES - 2020 · the central government. was the court language. The Hindus for long Statement 2 is correct: It introduced Dyarchy in provinces, under which

8 AIPTS 2020 (HIS - 16) (E) Answer Key Byju’s Classes: 9873643487

labour, healthy conditions of work, protection against the economic consequences of old

age, sickness. and unemployment.

28. Ans: (d) Explanation:

1. Bardoli Satyagraha: The Bardoli

Satyagraha, Feb 1928 was a movement in

the independence struggle led by Sardar

Vallabhai Patel for the farmers of Bardoli

against the unjust raising of taxes.

2. Nehru Report – 10 August 1928: The

Nehru Report had the primary motive of

assigning Dominion status to India within

the British Commonwealth. The major components of the Nehru Report are:

(a) Bill of Rights

(b) Assigning Equal rights to men and women

as citizens

(c) Formation of a federal form of government

with residuary powers in the hands of Centre

(d) Proposal for the creation of Supreme

Court

3. Karachi Resolution (26 to 31 March) was based upon the Gandhi Irwin pact of 5th

March 1931.

4. Gandhi-Irwin Pact, agreement signed on

March 5, 1931, between Mohandas K.

Gandhi, leader of the Indian nationalist

movement, and Lord Irwin (later Lord Halifax), British viceroy (1926–31) of

India. It marked the end of a period of

civil disobedience (satyagraha) in India

against British rule that Gandhi and his

followers had initiated with the Salt March (March–April 1930)

Karachi Resolution: The Karachi Congress

Session 1931 was presided over by Sardar

Vallabhbhai Patel. It was a special session of

Indian National Congress held at Karachi

from March 26 to 31 in 1931 to endorse the Gandhi Irwin Pact. Mahatma Gandhi was

nominated to represent the Indian National

Congress in the Second Round Table

Conference.

29. Ans: (b)

Explanation:

Option (a) is incorrect: India's first

maritime museum will be coming up at

Lothal in Gujarat. Option (b) is correct: Currently, the Indian

Space Research organisation (Isro) has two

launch pads at Sriharikotta, Andhra

Pradesh. Union Minister of State

(Independent Charge) Development of North-

Eastern Region (DoNER), MoS Atomic Energy

and Space, Dr Jitendra Singh in written reply to a question in Rajya Sabha said

"Government has proposal to set up rocket

launching pad near Kulasekarapattinam in

the state of Tamil Nadu". The development

comes on the backdrop of increasing launches from India, both for domestic and

international customers.

Option (c) is incorrect: Earliest epigraphic

evidence for sanskrit in South India was

discovered in Chebrolu village in Guntur

district of Andhra Pradesh. Option (d) is incorrect: The Chandrabhaga

beach in Puri district in the east Indian state

of Odisha has become the first in Asia to get

the Blue Flag certification.

30. Ans: (d)

Explanation:

Statement 1 is correct: Moderates had faith

in the British justice system. British rule to

most of them seemed to be an act of providence destined to bring in

modernisation. Indians needed some time to

prepare themselves for self-government; in

the meanwhile, absolute faith could be placed

in British parliament and the people. Their complaint was only against "un-British rule"

in India perpetrated by the viceroy, his

executive council and the Anglo-Indian

bureaucracy - an imperfection that could be

reformed or rectified through gentle

persuasion and by making them aware of the challenging conditions that ordinary Indian

face. They were conscious of the exploitative

nature of British rule, but wanted its reform,

not expulsion

Statement 2 is correct: The moderates were primarily influenced by Utilitarian theories,

as Edmund Burke, John Stuart Mill and

John Morley had left a mark on their

thoughts and actions. They believed that the

government should be guided by expediency

and not by any moral or ethical laws. And the constitution was to be considered inviolable

and hence repeatedly they appealed to the

British parliament complaining about the

Government of India subverting the

constitution. Statement 3 is correct: Some of the major

demands of moderates was that the budget

should be referred to the legislature, which

should have the right to discuss and vote on

it and also the right of interpellation. There

should also be a right to appeal to the Standing Committee of the House of

Commons against the Government of India.

Thus, their immediate demand was not for

full self-government or democracy; they

demanded democratic rights only for the

Page 9: ALL INDIA PRELIMS TEST SERIES - 2020 · the central government. was the court language. The Hindus for long Statement 2 is correct: It introduced Dyarchy in provinces, under which

9 AIPTS 2020 (HIS - 16) (E) Answer Key Byju’s Classes: 9873643487

educated members of the Indian society who would substitute for the masses.

31. Ans: (c)

Explanation: Statement 1 is incorrect: Satyendra Nath

Tagore, elder brother of Rabindra Nath Tagore

was the first Indian to qualify ICS exam in

1863. Surendra Nath Banerjee joined the

service in 1871 along with RC Dutt and

Behari Lal Gupta. Statement 2 is incorrect: Elected in 1913

to both the Bengal and imperial legislative

councils, Banerjee welcomed the principles of

the Montagu-Chelmsford report of 1918,

which recognized self-government as the goal of British policy in India

Statement 3 is correct: Surendra Nath

Banerjee was a moderate and opposed the

extreme methods advocated by the political

leader of BG Tilak and some of the non-

cooperation tactics that were practiced by Mahatma Gandhi.

32. Ans: (d)

Explanation:

Statement 1 is correct: Mahila Atma Raksha Samiti (MARS) was established

primarily to organize self defence by women

in the event of a Japanese aggression. MARS

came to depend heavily on their rural

constituents for a numerical show of strength. Apart from contributing to the

extension of the popular base of the

Communist Party in West Bengal, the MARS

acted as a powerful agent of change in the

politics of gender as well.

Statement 2 is correct: Latika Ghosh (1902-1983) was the daughter of the great poet

Monmohan Ghosh and niece of Aurobindo

Ghosh. When the Simon Commission landed

in Bombay in 1928, Latika Ghosh with more

than thousand ladies joined in the Hartal and Demonstration. It was Subhas Chandra Bose

who inspired Latika Ghosh in her activities

because he believed that the youth force of

Bengal could never be strengthened without

the help of women, especially young ladies.

Sm.Ghosh became the Secretary of "Mahila Rashtriya Sangha‖ and collected funds for

this Sangha.

Statement 3 is correct: The Rani of Jhansi

Regiment was the Women‘s Regiment of the

Indian National Army (INA) during the Second World War. It was named after Lakshmi Bai,

Rani of Jhansi, a revolutionary heroine. In

1944 the regiment was deployed in Burma

alongside other INA forces and the Japanese

army. 100 Rani troops are noted as forming a

vanguard unit as part of the ongoing Battle of Imphal.

33. Ans: (d)

Explanation:

The Union Home Ministry has altered States, warning them about the

vulnerability of the Android operating

system to a bug called 'StrandHogg'.

Promon, a Norwegian firm specialising in In-App protection, found proof of this

dangerous Android vulnerability, which

they call 'StrandHogg'.

It will allow cybercriminals to listen to the microphone, steal login credentials, take

photos using a camera, read SMS and

even access photos.

It basically exploits Android control settings called taskAffinity and

taskReparenting to allow apps including

malicious ones to freely assume the identity of another task in the

multitasking system.

It allows the malicious activity to hijack the target‘s task, so the next time the user

opens the target app, the hijacked tasks

will open up instead of the original tasks.

34. Ans: (c)

Explanation:

Statement 1 is correct: The Bardoli Taluk in

modern-day Gujarat was hit by floods and famines in 1925, which adversely affected

crop yield. This affected the farmers

financially. Ignoring the plight of the farmers,

the Bombay Presidency increased the tax

rates by 22%. Despite petitions and appeals

from civic groups and farmers to review this unjust hike in tax rates in lieu of the grave

situation, the government decided to go

ahead with tax collection. This situation led

to the unrest, eventually leading to

Satyagraha. Statement 2 is incorrect: The Hali system,

though was prevalent in south Gujarat, was a

speciality of Bardoli. This system is based on

the tribal agricultural labourers and the high

caste landlords, that is, Patidars. Relations

between a Hali labourer and the landowner who hired him were the same as those

between a serf and his master in medieval

feudal society. This system of recruiting

agricultural labour had grown out of money-

lending practices. The fact of the matter is that the Hali cultivated the lands owned by

upper caste people like Patidars, Anavil

Brahmins and also Rajputs. And the fact that

until 1938 no movement was launched for

freeing the agricultural serfs..

Page 10: ALL INDIA PRELIMS TEST SERIES - 2020 · the central government. was the court language. The Hindus for long Statement 2 is correct: It introduced Dyarchy in provinces, under which

10 AIPTS 2020 (HIS - 16) (E) Answer Key Byju’s Classes: 9873643487

Statement 3 is incorrect: The Bardoli Satyagraha of 1928, in the state of Gujarat,

India during the period of the British Raj, was

a major episode of civil disobedience and

revolt in the Indian Independence Movement.

The movement was eventually led by Vallabhbhai Patel, and its success gave rise

to Patel becoming one of the main leaders of

the independence movement. Neither

Mahatma Gandhi nor congress were directly

involved in the struggle. Kheda satyagraha

preceded the Bardoli movement, thus Bardoli satyagraha did not lay the foundations for

peasant resistance in Gujarat.

35. Ans: (c) Explanation:

Statement 1 is correct: According to the

Salt Act of 1882, only the British government

in India could manufacture, collect and sell

salt. This is called a monopoly. Actually, salt

can be manufactured by evaporating sea water, and many Indians living along the

coast have been doing so for ages. The

British, however, made this a criminal offence

and forced people to buy it at high prices.

Thus, Dandi march targeted salt manufacturing.

Statement 2 is incorrect: The Indian

National Congress, on 19 December 1929,

passed the historic ‗Purna Swaraj‘ (total

independence) resolution at its Lahore

session. A public declaration was made on 26 January 1930 – a day which the Congress

Party urged Indians to celebrate as

‗Independence Day‘. The declaration was

passed due to the breakdown of negotiations

between leaders of the freedom movement and the British over the question of dominion

status for India. It was only after the

declaration of purna swaraj that Mahatma

Gandhi decided to launch the Civil

Disobedience movement by breaking salt

law. Statement 3 is correct: On May 21, the poet

Sarojini Naidu led 2,500 marchers on the

Dharasana Salt Works, some 150 miles north

of Bombay. Several hundred British-led

Indian policemen met them and viciously beat the peaceful demonstrators. Matangini Hazra

(1869-1942) was an Indian revolutionary who

was shot dead by the British Indian police on

September 29, 1942. She was affectionately

known as Gandhi buri, Bangla for old lady

Gandhi. In 1932, she took part in the Non-Cooperation Movement and was arrested for

breaking the Salt Act.

36. Ans: (c) Explanation: The list of demands in the

ultimatum forwarded to Lord Irwin consisted

the following:

1. Prohibit intoxicants, 2. Change the ratio between the rupee and

the sterling,

3. Reduce the rate of land revenue,

4. Abolition of salt tax,

5. Reduce the military expenditure,

6. Reduce (Not increase) expenditure on civil administration,

7. Impose custom duty on foreign cloth,

8. Accept the Postal Reservation Bill,

9. Abolish the CID department,

10. Release all political prisoners, and 11. Issue licenses of arms to citizens for self-

protection.

Gandhi made it clear that if the 11 points are

ignored, the only way out was civil

disobedience. Breaking the salt laws of the government non-violently was the basic

activity of civil disobedience. Along with this

activity, activities like no tax campaign, no

revenue and no rent (land tax) campaign

became very popular in different parts of the country.

37. Ans: (c)

Explanation:

Statement 1 is correct: According to the Committee ‗wastage‘ meant premature

withdrawal of children from school at any

stage before the completion of the primary

course. By ‗stagnation‘ the committee meant

detention in the same classes for more than

one academic year. The committee had highlighted the following causes of wastage

and stagnation in primary education.

Statement 2 is incorrect: The committee felt

that there was unhealthy competition among

the universities. They paid more attention to increasing the number of students than to

raising the standard of education.

Statement 3 is incorrect: In order to remove

the defects of the system of secondary

education the committee made the following

recommendations— Diverting Pupils to Non-Literary Pursuits with a view to reduce the

domination of the matriculation examination,

the committee recommended— a) The

introduction of a more diversified curriculum

in the middle vernacular schools, b) The diversion of more boys to industrial and

commercial careers at the end of the middle

stage.

Page 11: ALL INDIA PRELIMS TEST SERIES - 2020 · the central government. was the court language. The Hindus for long Statement 2 is correct: It introduced Dyarchy in provinces, under which

11 AIPTS 2020 (HIS - 16) (E) Answer Key Byju’s Classes: 9873643487

38. Ans: (c) Explanation:

Option (a) is correct: The Indian National

Congress was the only organisation which

put up candidates on an all-India basis. Option (b) is correct: From the outset of the

election campaign, the administration did its

best to prevent Congress successes wherever

possible. Methods of intimidation,

interference, repression and banning of

meetings were employed.In the North-West Frontier Province the Congress was declared

an illegal organisation, and its leader, Khan

Abdul Gaffar Khan, refused permission to

return to his home. Despite this, the

Congress succeeded in winning 19 seats out of a total of 50.

Option (c) is incorrect: The election results

were a blow to the Muslim League. All India

Muslim League won 106 seats (6.7% of the

total), placing it as second ranking party. The

poor showing by the Muslim League in the elections was neither surprising nor

unexpected. The League was essentially an

urban-based political party and had little or

no contact with the masses in 1937.

Congress had clear majority in five provinces i.e. United Provinces (U.P.), Bihar, Madras,

Central Provinces (C.P.), Orissa. In Bengal,

NWFP, Assam and Bombay, Congress

emerged as the single largest party. It was in

Punjab and Bengal that Muslim League

formed coalition governments, though it was not able to form independent government in

any of the 11 provinces.

Option (d) is correct: In order to coordinate

all elements who desired to contest the

elections on the Congress ticket, and facilitate the election campaign, the Congress

established the All-India Congress

Parliamentary Board. The duty of this Board

was to select from among the prospective

candidates nominated for a constituency the

person who would be endorsed as the Congress candidate.

39. Ans: (c)

Explanation: Social Awareness and Actions

to Neutralize Pneumonia Successfully (SAANS) is an initiative under the National

Health Mission. It is to accelerate action to

reduce deaths due to Childhood Pneumonia.

Under this, the government is targeting a

reduction in pneumonia-caused deaths by 2025 to less than 3 deaths out of 1000 live

births.

40. Ans: (a) Statement 1 is correct: Congress had a

British committee based in London, acting as

a lobby group in Britain, which was founded

in 1889. Dadabhai Naoroji, when he was an MP in London, attended this group's

meetings, and was associated with their

parliamentary pressure group. In 1890, the

committee began to publish India, a free

monthly journal summarising Indian news for

the British press and politicians. India became a weekly subscribed journal.

Statement 2 is correct: Gandhiji had

resigned from membership of the Congress in

October 1934 and devoted himself mainly to

his "constructive programme," but continued to be consulted by the leaders of the

Congress. He travelled through India,

teaching ahimsa and demanding eradication

of ‗untouchability‘.

Statement 3 is incorrect: In 1934 Lohia

became actively involved in the Congress Socialist Party (CSP), founded that year as a

left-wing group within the Indian National

Congress; he served on the CSP executive

committee and edited its weekly journal.

41. Ans: (d)

Explanation:

Pair 1 is correctly matched: Kanaklata

Barua was a freedom fighter and martyr. In

1942 when the Non Cooperation Movement was at its height, Kanaklata was in Tezpur.

She joined the movement and became

involved with the setting up of the secret

Death Squads (Mrityu Bahini). On 20

September 1942, just a few months short of

her eighteenth birthday, she joined the column of protesters with a tricolour flag in

her hand. She was shot in the chest while

trying to hoist the national flag at the Gohpur

police station as part of the Death Squad

programme. She died instantly. Kanaklata immediately became an icon for the freedom

fighters of India.

Pair 2 is correctly matched: Raj Kumari

Gupta was born about a century ago in the

little-known Banda zilla of Kanpur, she and

her husband worked closely with Mahatma Gandhi and Chandrashekhar Azad. Her

crucial contribution to the Kakori dacoity

case barely figures in the narratives of

freedom. Raj Kumari, who was given the

charge of supplying revolvers to those involved in the Kakori operation, apparently

hid the firearms in her undergarment and set

out in khadi clothes to deliver them, with her

three-year-old son in tow. On being arrested,

she was disowned by her husband‘s family

and thrown out of her marital home.

Page 12: ALL INDIA PRELIMS TEST SERIES - 2020 · the central government. was the court language. The Hindus for long Statement 2 is correct: It introduced Dyarchy in provinces, under which

12 AIPTS 2020 (HIS - 16) (E) Answer Key Byju’s Classes: 9873643487

Pair 3 is correctly matched: Tara Rani was born in Saran near Patna. At a very early age,

she got married to a freedom fighter named

Phulendu Babu and actively participated in

protest marches against the British regime.

Tara gathered like-minded women to carry out demonstrations during the Quit India

Movement.

Pair 4 is correctly matched: Rani Gaidinliu

was born on January 26, 1915 at Nungkhao,

a Rongmei village in Manipur. She was 16

when she was the leader of the Heraka movement. While the Heraka movement was

long aware of the civil disobedience

movement in British India, it was Gaidinliu

who first used Gandhiji‘s name and identified

her peoples‘ struggle against oppression and self-determination with the larger national

movement gaining ground in India. Through

armed resistance, she quickly transformed a

religious-indigenous rebellion into a

revolutionary movement for independence. In

1932, she was tried and convicted on a charge of murder, waging war against the

British crown and sentenced to life

imprisonment. When Nehru met Gaidinliu,

she had already been imprisoned for five

years. Nehru‘s efforts and subsequent failure to secure Gaidinliu‘s release from the British

is well documented. She would remain a

prisoner for another decade, before being

released from Tura Jail on October 14, 1947,

after India became independent.

42. Ans: (c)

Explanation:

Statement 1 is incorrect: The first INC

session to be held in a village was Faizpur session of 1937.

Statement 2 is correct: Resolution for

‗Poorna Swaraj‘ was passed in 1929 Lahore

session of INC presided over by Jawaharlal

Nehru. The goal of purna swaraj was

reiterated in the karachi session too. Statement 3 is correct: The Karachi session

of 1931 was presided over by Sardar

Vallabbhai Patel. The session was famous for

the passing of Resolution on Fundamental

rights and Economic Policy. Some important aspects of these resolutions were: Basic civil

rights of freedom of speech, Freedom of Press,

Freedom of assembly, Freedom of association,

Equality before law Elections on the basis of

Universal Adult Franchise Free and

compulsory primary education. Substantial reduction in rent and taxes. Better conditions

for workers including a living wage, limited

hours of work. Protection of women and

peasants Government ownership or control of

key industries, mines, and transport and

Protection of Minorities. The Economic Policy included the provisions of protection of

domestic industries against foreign capital.

Regulation of currency in national interest.

Relief from agricultural indebtedness and

usury and the state ownership of key industries.

Statement 4 is correct: The Gandhi Irwin

Pact was endorsed by the Congress in the

Karachi Session of 1931.

This session also nominated Mahatma

Gandhi to represent Congress in the Second Round Table Conference.

43. Ans: (d)

Explanation: The US and India have announced the establishment of an annual

Parliamentary Exchange as a part of India-US

2+2 Ministerial dialogue. It will discuss issues

of strategic importance to the bilateral

relationship. It will feature members in the

US and Indian parliamentarians travelling to each other's countries every other year to

discuss issues of strategic importance to the

bilateral relationship.

44. Ans: (b) Explanation:

Statement 1 is correct: To gain the Indian

support in the British war efforts, Cripps

Mission was sent to India in March 1942.

Congress demanded the immediate transfer of effective power to Indians, which was

refused. Thus, Congress decided to launch

the movement.

Statement 2 is incorrect: The All India

Congress Committee met at Bombay on 8

August 1942 to pass the Quit India Resolution and proposed the starting of a

non-violent mass struggle under Gandhi‘s

leadership. But the movement was not non-

violent.

Statement 3 is correct: During the agitation parallel governments were set by

revolutionaries in Ballia in Eastern U.P,

Tamluk in Midnapur district of Bengal and

Satara district of Bombay.

45. Ans: (b) Explanation:

Statement 1 is correct: The Muslim League

opposed the Quit India Movement. Thus there

was a gap in the nationalist‘s efforts towards the freedom struggle. C. Rajagopalachari, the

veteran congress leader prepared a formula

for Congress-League cooperation in 1944.

Statement 2 is correct: After the end of the

Second World War, the entire population of

Muslim majority areas in the North-West and

Page 13: ALL INDIA PRELIMS TEST SERIES - 2020 · the central government. was the court language. The Hindus for long Statement 2 is correct: It introduced Dyarchy in provinces, under which

13 AIPTS 2020 (HIS - 16) (E) Answer Key Byju’s Classes: 9873643487

North-East India to decide by a plebiscite, whether or not to form a separate sovereign

state.

Statement 3 is incorrect: In 1944, Gandhi

and M A Jinnah held talks on the basis of the

Rajaji Formula. But the talks were a failure as Jinnah had objections to the proposal and

he wanted the INC to accept the Two-Nation

Theory.

46. Ans: (a) Explanation:

Statement 1 is incorrect: The Lahore

resolution of the Muslim League in March

1940 formally proclaimed the Muslims as a

nation. It did not mention partition or Pakistan, but only talked about "Independent

states" to be constituted of the Muslim

majority provinces in an unspecified "future".

The resolution, in other words, only signalled

the transformation of Indian Muslims from a

minority to a nation so that no future constitutional arrangement for India could

any more be negotiated without their

participation and consent.

Statement 2 is incorrect: In 1946,

Muhammad Ali Jinnah declared 16 August as ‗Direct Action Day‘ and called for Muslims all

over the country to ‗suspend all business‘.

This was to put pressure on the British

government to relent to the Muslim League‘s

demand of dividing the country on the basis

of religion, thereby allowing the creation of a Muslim-dominated Pakistan. It is not related

to the wavell plan of June 1945. Failure of

cabinet mission plan led to immediate

announcement of Direct Action Day by

Muslim League. Statement 3 is correct: Shimla conference

was a failure because the League and the

Congress could not settle their differences.

Jinnah insisted that only Muslim League

members could be the Muslim

representatives in the Council, and opposed to the Congress nominating Muslim

members. This was because Jinnah wanted

the League to be the sole representative of

Muslims in India.

47. Ans: (c)

Explanation: In August 1942, Gandhiji

started the 'Quit India Movement' and

decided to launch a mass civil disobedience

movement 'Do or Die' call to force the British to leave India.

Statement 1 is incorrect: It initially started

as an urban revolt, marked by strikes,

boycott and picketing, which were quickly

suppressed. In the middle of August, the

focus shifted to the countryside, which

witnessed a major peasant rebellion, marked by destruction of communication systems,

such as railway tracks and stations,

telegraph wires and poles, attacks on

government buildings or any other visible

symbol of colonial authority and finally, the formation of "national governments" in

isolated pockets.

Statement 2 is correct: During the

movement, a clandestine radio station was

run by Usha Mehta, along with her

associates, Vithalbhai Jhaveri, Chandrakant Jhaveri, Babubhai Thakkar and Nanka

Motwani, (the owner of Chicago Radio, and

the person who supplied equipment and

provided technicians). The radio broadcasted

messages from Gandhi and other prominent leaders across the country.

Statement 3 is correct: Not only the

educated youth participated in such

activities, but also bands of ordinary

peasants organised such subversive actions

by night, which came to be known as the "Karnataka method". Thus part time peasant

squads engaged in farming by day and

sabotage activities at night was known as the

Karnataka method and it is associated with

QIM.

48. Ans: (c)

Explanation: Operation Twist is the name

given to a U.S Federal Reserve monetary

policy operation in 2011-12 that involves the purchase and sale of bonds. RBI have

recently decided to launch India‗s version of

Operation Twist. Under this, RBI will

simultaneously buy and sell government

securities worth Rs. 10,000 crore each to ease interest rates for long-term government

bonds. RBI will purchase the longer-term

government bonds maturing in 2029 at

6.45% and simultaneously sell short term

bonds maturing in 2020. Open Market

Operations (OMO) is one of the quantitative (to regulate or control the total volume of

money) monetary policy tools which is

employed by the central bank of a country to

control the money supply in the economy.

49. Ans: (c)

Explanation: The Interim Government of

India, also known as the Provisional

Government, formed on 2 September 1946

from the newly elected Constituent Assembly of India, had the task of assisting the

transition of British India to independence.

Statement 1 is incorrect: Viceroy Wavell

managed to constitute an Indian interim

government without the Muslim League. A

Congress dominated government was sworn

Page 14: ALL INDIA PRELIMS TEST SERIES - 2020 · the central government. was the court language. The Hindus for long Statement 2 is correct: It introduced Dyarchy in provinces, under which

14 AIPTS 2020 (HIS - 16) (E) Answer Key Byju’s Classes: 9873643487

in on 2 September 1946 with JawaharlaJ Nehru as the prime minister. It was

constituted based on the cabinet mission

plan.

Statement 2 is correct: In late October the

Muslim League was also persuaded to join and finally League joined the Indian interim

government.

Statement 3 is incorrect: Ambedkar was

not a member of the interim government,

later he was joined as a member of the

constituent assembly.

50. Ans: (a)

Explanation: Lord Mountbatten was

assigned the task of a speedy transfer of power by the then British Prime Minister

Clement Atlee. In May 1947, Mountbatten

came up with a plan under which he

proposed that the provinces be declared

independent successor states and then be

allowed to choose whether to join the constituent assembly or not. This plan was

called the ‗Dickie Bird Plan‘. Jawaharlal

Nehru, when appraised of the plan,

vehemently opposed it saying it would lead to

balkanisation of the country. Hence, this plan was also called Plan Balkan.

Statement 1 and 2 are correct: Lord

Mountbatten under the Plan Balkan,

proposed the partition of Punjab and Bengal

and handing over power to the provinces and

sub-provinces, which would be free to join one or more of group Constituent Assemblies

on the basis of self-determination.

Statement 3 is incorrect: Nehru rejected

these proposals on the grounds that instead

of producing any sense of certainty, security and stability, they would encourage

disruptive tendencies everywhere and chaos

and weakness. Jinnah cast them aside too, as

he was not yet prepared to accept the

partition of Punjab and Bengal which would

give him only a "truncated or mutilated, moth-eaten Pakistan.

51. Ans: (c)

Explanation: The Anarchical and

Revolutionary Crimes Act of 1919, popularly known as the Rowlatt Act or Black Act, was a

legislative act passed by the Imperial

Legislative Council in Delhi on 21 March

1919, indefinitely extending the emergency

measures of preventive indefinite detention, incarceration without trial and judicial review

enacted in the Defence of India Act 1915

during the First World War.

Statement 1 is incorrect: Mahatma Gandhi

organised Satyagraha Sabha and he himself

became its President. The Satyagraha Sabha

published propaganda literature and collected signatures for a Satyagraha pledge.

Statement 2 is correct: During the passage

of Rowlatt Act many non-official Indian

members of Imperial Legislative Council

opposed the act, but interestingly, some of the Nationalist like DE Wacha, Surendra

Nath Banerjee, Tej Bahadur Sapru and

Srinivas Sastri were against the Satyagraha.

Statement 3 is correct: During the

Satyagraha the government clamped down

heavily on the people. There were violent clashes in many parts. While the hartal was

successful in Delhi, Punjab and a few other

places witnessed violence. In the wake of the

violence, the hartal was suspended by

Gandhi. The protests were very intense in Punjab. Two Congress leaders Dr. Satya Pal

and Dr. Saifuddin Kitchlew were arrested.

The army was deployed in Punjab where

martial law was enacted.

52. Ans: (b) Explanation: The Citizenship Amendment

Act (CAA), 2019 was recently passed by the

Parliament.

Statement 1 is correct: The amendment provides that those illegal migrants who fulfil

following four conditions will not be treated

as illegal migrants.

They are from Afghanistan, Bangladesh or Pakistan

They are Hindus, Sikhs, Buddhists, Jains, Parsis or Christians

They entered India on or before December 31, 2014

They did not reside in certain tribal areas of Assam, Meghalaya, Mizoram, or

Tripura included in the Sixth Schedule to

the Constitution or areas under the ―Inner

Line‖ permit, in Arunachal Pradesh,

Mizoram, and Nagaland.

Statement 2 is incorrect: It empowers the

central government to cancel registration of

Overseas Citizen of India, if they show

disaffection to the Constitution or they

engage with the enemy during war or in the

interest of sovereignty of India and security of state.

Statement 3 is incorrect: The act is not

applicable in certain tribal areas of Assam,

Meghalaya, Mizoram, or Tripura included in

the Sixth Schedule to the Constitution or areas under the ―Inner Line‖ permit, in

Arunachal Pradesh, Mizoram, and Nagaland.

The above areas are not included in the fifth

schedule of the Constitution.

Page 15: ALL INDIA PRELIMS TEST SERIES - 2020 · the central government. was the court language. The Hindus for long Statement 2 is correct: It introduced Dyarchy in provinces, under which

15 AIPTS 2020 (HIS - 16) (E) Answer Key Byju’s Classes: 9873643487

53. Ans: (a) Explanation: The Khilafat movement, also

known as the Indian Muslim movement

(1919–24), was a pan-Islamist political

protest campaign launched by Muslims of British India to restore the caliph of the

Ottoman Caliphate. It was a protest against

the sanctions placed on the khalifa and the

Ottoman Empire after the First World War by

the Treaty of Sèvres.

Statement 1 is incorrect: The khilafat and non cooperation movement brought the

urban muslims into the national movement.

There were many Urban educated Muslim

who participated like Muhammad Ali,

Maulana Abul Kalam Azad,Dr Hakim Ajmal Khan etc

Statement 2 is correct: The support of the

Congress was quite essential for the Khilafat

movement to succeed. However, although

Gandhi was in favour of launching

Satyagraha and non cooperation against the government on the Khilafat issue the

Congress was not United on this form of

political action. Bal Gangadhar Tilak was

opposed to having an Alliance with Muslim

leaders over a religious issue and he was also sceptical for Satyagraha as an instrument of

politics. There was also opposition to some of

the Other provisions of Mahatma Gandhi's

non cooperation program such as boycott of

the councils.

Statement 3 is correct: Mohammad Ali and his brother Maulana Shaukat Ali joined with

other Muslim leaders to form the All India

Khilafat Committee. The organisation was

based in Lucknow, India. They aimed to build

political unity amongst Muslims and use their influence to protect the caliphate. In

1920, they published the Khilafat Manifesto,

which called upon the British to protect the

caliphate and for Indian Muslims to unite and

hold the British accountable for this purpose.

54. Ans: (d)

Explanation: The Second Round Table

Conference was to be held in 1931 in London.

In 1930, the Salt Satyagraha was conducted

and India and Gandhi received worldwide attention. The British government in India

was criticised for its unjust treatment of

Indians. Gandhi and many other leaders were

imprisoned along with thousands of Indians.

Lord Irwin wanted the issue to come to an end. So, Gandhi was released from prison in

January 1931. Accordingly, Gandhi met Irwin

and held negotiations. It was for the first time

that the two were meeting as ‗equals‘.

Statement 1 is correct: The Pact also agreed to restore the confiscated properties of the

Satyagrahis.

Statement 2 is incorrect: In the Gandhi –

Irwin Pact – some of the Demands of Gandhi

not agreed to by Irwin:

A public inquiry into police excesses during its suppression of the Civil

Disobedience Movement.

Commuting the death sentences of Bhagat Singh and his associates to life

sentences.

Statement 3 is correct: The then Congress President Sardar Vallabhai Patel authorised

Gandhi to hold talks with Lord Irwin.

Accordingly, Gandhi met Irwin and held

negotiations.

55. Ans: (c)

Explanation: The Royal Indian Navy revolt

(also called the Royal Indian Navy mutiny or

Bombay mutiny) encompasses a total strike

and subsequent revolt by Indian sailors of the Royal Indian Navy on board ship and shore

establishments at Bombay harbour on 18

February 1946.

Statement 1 is correct: The RIN Revolt

started as a strike by ratings of the Royal

Indian Navy on 18 February in protest against general conditions. The immediate

issues of the revolt were living conditions and

food. They also included other National issues

like release of INA prisoners and withdrawal

of Indian troops from Indonesia. Statement 2 is incorrect: The mutineers in

the armed forces got no support from the

national leaders and were largely leaderless.

Mahatma Gandhi, in fact, condemned the

riots and the ratings' revolt. His statement on

3 March 1946 criticized the strikers for revolting without the call of a "prepared

revolutionary party" and without the

"guidance and intervention" of "political

leaders of their choice. "The Muslim League

issued similar attacks on the mutiny which argued that the unrest of the sailors was not

best expressed on the streets, however

serious the grievance may be. The

Communist Party of India, the third largest

political force at the time, extended full

support to the naval ratings and mobilised the workers in their support, hoping to end

British rule through revolution rather than

negotiation.

Statement 3 is correct: The revolt was

called off following a meeting between the President of the Naval Central Strike

Committee (NCSC), M. S. Khan, and Sardar

Page 16: ALL INDIA PRELIMS TEST SERIES - 2020 · the central government. was the court language. The Hindus for long Statement 2 is correct: It introduced Dyarchy in provinces, under which

16 AIPTS 2020 (HIS - 16) (E) Answer Key Byju’s Classes: 9873643487

Vallabhai Patel of the Congress, who had been sent to Bombay to settle the crisis. Patel

issued a statement calling on the strikers to

end their action, which was later echoed by a

statement issued in Calcutta by Mohammed

Ali Jinnah on behalf of the Muslim League. Under these considerable pressures, the

strikers gave way.

56. Ans: (d)

Explanation: The Simla Conference 1945 was a meeting between the Viceroy of India

Lord Wavell and the major political leaders of

British India at Simla. Convened to agree on

and approve the Wavell Plan for Indian self-

government, and there it reached a potential agreement for the self-rule of India that

provided separate representation for Muslims

and reduced majority powers for both

communities in their majority regions. On 14

June 1945, Lord Wavell announced a plan for

a new Executive Council in which all members except the Viceroy and the

Commander in Chief would be Indians.

Statement 1 is correct: The Plan proposed;

The Viceroy‘s Executive Council would be immediately reconstituted and the

number of its members would be increased.

In the Council there would be equal representation of high-caste Hindus and

Muslims.

The Viceroy would convene a meeting of Indian politicians including the leaders of

Congress and the Muslim League at

which they would nominate members of the new Council.

The Wavell Plan, in essence, proposed the

complete Indianisation of the Executive

Council, but instead of asking all the parties

to nominate members to the Executive

Council from all the communities, seats were reserved for members on the basis of religion

and caste, with the caste Hindus and

Muslims being represented on it on the basis

of parity.

Statement 2 is correct: The reconstructed

Council was to function as an interim

government till a Permanent solution was

arrived at.

Statement 3 is incorrect: An Indian would

be appointed as the member for Foreign Affairs in the Council. However, a British

commissioner would be responsible for trade

matters. The defence of India would remain in

British hands until power was ultimately

transferred to Indians.

Statement 4 is correct: If this plan were to be approved for the central government, then

similar councils of local political leaders

would be formed in all the British Indian

provinces.

57. Ans: (d)

Explanation: Following the tragedy of

December 2012, the Government has set up a

dedicated fund – Nirbhaya Fund – which can

be utilized for projects specifically designed to improve the safety and security of women.

Statement 1 is correct: It provides for a

non-lapsable corpus fund to support

initiatives of government and NGOs.

Statement 2 is incorrect: Central Victim Compensation Fund (CVCF) has been funded

under the Framework of Nirbhaya Fund to

support States/ UTs for their Victim

Compensation Scheme. The total cost of the

project is Rs.200.00 Cr. The CVCF is a one-

time grant of top-up funds to the States/UTs subsequent to the directions of Hon‘ble

Supreme Court. Funds were released to

States/UTs in the year of 2016-17. No further

activity remains on part of the central

government Statement 3 is correct: The Ministry of

Women and Child Development is the nodal

authority to appraise schemes under the

Nirbhaya Fund.

58. Ans: (a) Explanation: Subhas Chandra Bose was one

of the most eminent freedom fighters of India.

Statement 1 is incorrect: Bose joined the

Indian National Congress (INC) in 1921. He

was the President of the All India Youth Congress and also the Secretary of the Bengal

State Congress. He advocated complete

Swaraj and was in favour of the use of force

to gain it.

He had differences with Gandhi and he wasn‘t keen on non-violence as a tool for

independence. Bose stood for and was elected

the party‘s president in 1939 but was forced

to resign due to differences with Gandhi‘s

supporters. Bose‘s ideology tilted towards

socialism and leftist authoritarianism. He formed the All India Forward Bloc in 1939 as

a faction within the Congress. It was not a

parallel organisation rather worked within the

Congress to propagate its own ideology.

Statement 2 is correct: INA found support among expatriate Indians and under its aegis

Bose formed the Azad Hind government

which came to produce its own currency,

postage stamps, court and civil code. It was

recognised by Axis states.

Page 17: ALL INDIA PRELIMS TEST SERIES - 2020 · the central government. was the court language. The Hindus for long Statement 2 is correct: It introduced Dyarchy in provinces, under which

17 AIPTS 2020 (HIS - 16) (E) Answer Key Byju’s Classes: 9873643487

Statement 3 is correct: Rash Behari Bose handed over INA to Subhas Chandra Bose. It

was revived under the leadership of Subhas

Chandra Bose after his arrival in Southeast

Asia in 1943. The army was declared to be

the army of Bose's Arzi Hukumat-e-Azad Hind (the Provisional Government of Free

India). Under Bose's leadership, the INA drew

ex-prisoners and thousands of civilian

volunteers from the Indian expatriate

population in Malaya (present-day Malaysia)

and Burma. This INA under the leadership of Subash Bose fought along with the

Imperial Japanese Army against the British

and Commonwealth forces in the campaigns

in Burma: at Imphal and Kohima, and later

against the Allied retaking of Burma.

59. Ans: (c)

Explanation: Lord Mountbatten (India‘s last

viceroy) proposed a plan in May 1947

according to which provinces were to be declared independent successor states with

the power to choose whether to join the

constituent assembly or not. On July 18,1947

the British Parliament ratified the

Mountbatten plan as Indian Independence Act. It was accepted by the Congress and the

Muslim League.

Statement 1 is correct: British India was to

be partitioned into two dominions – India and

Pakistan. It empowered the constituent

assembly of the two Dominions to frame and adopt any constitution for their respective

Nations and to repeal any act of British

Parliament including the Independence Act

itself.

Statement 2 is incorrect: The princely states were given the choice to either remain

independent or accede to India or Pakistan.

The British suzerainty over these kingdoms

was terminated.

Statement 3 is correct: The act abolished

the office of Viceroy and provided for each dominion of India and Pakistan, a Governor

General, who was to be appointed by British

King on the advice of the dominion cabinet. It

deprived the British monarch of its right to

withdraw or ask for reservation of certain bills for its approval but this right was

reserved for the governor general. Governor

General would have full power to assent to

any bill in the name of his majesty. The

Governor-general and Provincial Governors

were made constitutional heads of State. They were made to act on the advice of

respective councils of Ministers in all

matters.

60. Ans: (a) Explanation: ‗Rashtriya Vayoshri Yojana‘

(RVY) is being implemented with an objective

to provide to senior citizens, belonging to BPL

category and suffering from age-related disabilities/ infirmities, with such physical

aids and assisted living devices which can

restore near normalcy in their bodily

functions.

Statement 1 is correct: It is a scheme for

providing Physical Aids and Assisted-living Devices for Senior citizens belonging to the

BPL category.

Statement 2 is correct: It is a Central Sector

Scheme launched by the Ministry of Social

Justice and Empowerment in 2017. The Scheme is implemented through the Artificial

Limbs Manufacturing Corporation (ALIMCO).

Under the Scheme, assisted living devices

such as Walking Sticks, Elbow Crutches,

Walkers/ Crutches, Hearing Aids,

Wheelchairs, Spectacles, etc. are provided free of cost to the beneficiary senior citizens.

Statement 3 is incorrect: The State

Governments/ UT Administrations identifies

the Beneficiaries in each district through the

Committee chaired by the Deputy Commissioner/District Collector. The State

Government/UT Administration/District

Level Committee utilises the data of BPL

beneficiaries receiving Old Age Pension under

the NSAP or any other Scheme of the

State/UT for identification of senior citizens belonging to BPL category.

61. Ans: (d)

Explanation:

Option (a) is incorrect: After the Indian Revolt of 1857, it was tactfully decided to

provide some non-official representation on

the Governor General's and the Governors

councils. The Indian councils Act of 1861

were the result of this decision, providing just non official representation to native

population.

Option (b) is incorrect: Even the word

election was not mentioned under the act. A

few of the non-official seats were still to be

filled by simple nomination, but for a majority of them 'recommendations' were to be made

by the local bodies or corporations, religious

communities, municipalities, universities,

chambers of commerce and the like.

Option (c) is incorrect: The Indian Councils Act 1909 popularly known as the Morley

Minto Reforms of 1909 further enlarged the

legislative councils both of the Governor

General and that of the provinces. They also

introduced, for the first time, the method of

election, though indirect, as the means of

Page 18: ALL INDIA PRELIMS TEST SERIES - 2020 · the central government. was the court language. The Hindus for long Statement 2 is correct: It introduced Dyarchy in provinces, under which

18 AIPTS 2020 (HIS - 16) (E) Answer Key Byju’s Classes: 9873643487

constituting a portion of the non-official members.

Option (d) is correct: After Montagu

Chlemsford reforms were proposed, the

Government of India decided in favour of

direct election for both the Houses of the Central Legislature thus, the majority of the

members of the Legislative Council taken

together, they formed 77.8% of the total

number.

62. Ans: (b) Explanation: To get Indian cooperation in the

second World War effort, the then viceroy

Linlithgow announced the August Offer

(August 1940). It proposed-

Dominion status as the objective for India.

Expansion of viceroy‘s executive council.

Setting up of a constituent assembly after the war where mainly Indians would

decide the constitution according to their

social, economic and political

conceptions, subject to fulfillment of the

obligation of the Government regarding defence, minority rights, treaties with

states, all India services.

Statement 1 is incorrect: August Offer was

offered by the Viceroy Linlithgow, and not

Lord Wavell. Statement 2 is correct: It was for the first

time, that the right of Indians to frame their

own constitution and formation of constituent

assembly was accepted.

63. Ans: (c)

Explanation:

Statement 1 is correct: Poona Pact, (Sept.

24, 1932), agreement between Hindu leaders

in India granting new rights to untouchables (low-caste Hindu groups). The pact, signed at

Poona (now Pune, Maharashtra), resulted

from the communal award of Aug. 4, 1932,

made by the British government on the

failure of the India parties to agree, which

allotted seats in the various legislatures of India to the different communities. Mahatma

Gandhi objected to the provision of separate

electorates for the Scheduled (formerly

―untouchable‖) Castes, which in his view

separated them from the whole Hindu community.

Statement 2 is correct: Pact abandoned

separate electorate for depressed class but

seats reserved for the classes were increased

from 71 to 147 in provincial legislature and

18 percent of total in Central legislature.

64. Ans: (c) Explanation:

Statement 1 is correct: All India Trade

Union Congress (AITUC) was formed in 1920

to elect an Indian delegation to the International Labour Organization. AITUC is

the first Central Trade Union of India. The

first session of the AITUC had Lala Lajpat Rai

as its President. AITUC is affiliated to the

World Federation of Trade Unions (WFTU) a

major International Trade Union which has affiliates all over the world. In its second

session in 1921 in Jharia had adopted a

resolution of Swaraj (Complete independence

from British rule), almost eight years before

the platform of freedom struggle- the Indian National Congress adopted such resolution in

1929.

Statement 2 is correct: The Non-

cooperation resolution adopted by congress in

1920 talked about oppression of workers by

foreign agents only.

65. Ans: (b)

Explanation: Partial Credit Guarantee

Scheme was issued on 10.8.2019.

Statement 1 is incorrect: The Scheme was issued for providing one-time partial credit

guarantee by Government to Public Sector

Banks (PSBs) for purchase of assets by them

from NBFCs / HFCs, limited to 10 percent of

fair value of assets purchased by the banks under the Scheme of Rs.10,000 crore,

whichever is lower. The window was for a

period of six months from the date of

issuance of the Scheme or till such date by

which Rs.1,00,000 crore of assets get

purchased by the Banks, whichever is earlier. Statement 2 is incorrect: Power has been

delegated to the Finance Minister to extend

the validity of the Scheme by up to three

months taking into account its progress.

Statement 3 is correct: The scheme aims to address temporary asset liability mismatches

of otherwise solvent NBFCs/HFCs without

having to resort to distress sale of their assets

to meet their commitments.

66. Ans: (a) Explanation:

Statement 1 is incorrect: A series of Praja

Mandals were established to promote the

nationalist creed in the Princely states. The people of the Praja mandal movement fought

against their feudal princes and British

administration simultaneously. They

demanded democratic rights. They

implemented constructive programs of the

Congress in princely states. They established

Page 19: ALL INDIA PRELIMS TEST SERIES - 2020 · the central government. was the court language. The Hindus for long Statement 2 is correct: It introduced Dyarchy in provinces, under which

19 AIPTS 2020 (HIS - 16) (E) Answer Key Byju’s Classes: 9873643487

schools, encouraged cottage industries and started agitation against untouchability.

Statement 2 is correct: These Mandals were

eventually affiliated to a national body called

the All India States' People's Conference,

founded in 1927 with its headquarters at Bombay. It raised moderate demands for

democratic rights and constitutional changes,

to which many of the princes responded with

sharp vengeance and massive repression.

67. Ans: (b) Explanation:

Statement 1 is incorrect: Muslim and

Hindu Maha sabha had fared poorly in the

1937 elections. The communalists now realized that they would gradually wither

away if they did not take to militant, mass-

based politics. Hitherto, organized mass

movements and cadre-based politics had

been built by radical, anti-status quo

nationalists. Moreover, the Congress had not yet acquired firm roots among all the masses,

especially among the Muslim masses. Thus,

the Congress decided to initiate, under

Jawaharlal Nehru‘s guidance, a massive

campaign to work among the Muslim masses, known as the Muslim Mass Contact

Programme.

Statement 2 is correct: The Muslim League

headed by Jinnah observed ‗Day of

Deliverance‘ on 22 December 1939 when the

Congress Party members who were part of the Central and provincial governments resigned

in mass protesting the Viceroy‘s decision to

make India a party to the Second World War

without duly consulting Indians.

68. Ans: (c)

Explanation:

Statement 1 is correct: Sir Sayyid started a

modernisation movement among the Muslims

and founded for this purpose the Mohammedan Anglo-Oriental College in

Aligarh in 1875.

Statement 2 is correct: Theodore Beck, the

European principal of the Aligarh College,

who formed in 1888 the Indian Patriotic

Association to oppose Congress and to plead for government patronage for the Muslims. In

1893 the Mohammedan Anglo-Oriental

Defence Association was formed, once again

with Beck's encouragement, to check the

growing popularity of the Congress and to organise Muslim public opinion against it.

69. Ans: (b) Explanation:

Statement 1 is correct: Scheme will promote

panchayat led ground water management and

behavioural change with primary focus on demand side management.

Statement 2 is incorrect: Scheme has been

designed with the principal objective of

strengthening the institutional framework for

participatory groundwater management and

bringing about behavioral changes at the community level for sustainable groundwater

resource management in seven States, viz.

Gujarat, Haryana, Karnataka, Madhya

Pradesh, Maharashtra, Rajasthan and Uttar

Pradesh. Statement 3 is incorrect Out of the total

outlay of Rs. 6000 crore to be implemented

over a period of 5 years (2020-21 to 2024-25),

50% shall be in the form of World Bank loan,

and be repaid by the Central Government.

The remaining 50% shall be through Central Assistance from regular budgetary support.

The entire World Bank's loan component and

Central Assistance shall be passed on to the

States (not directly to the Panchayats)as

Grants.

70. Ans: (d)

Explanation:

Pair 1 is correctly matched: Dadabhai

Naoroji started the newspaper voice of India. In 1883 he started the Voice of India in

Bombay and later incorporated it into the

Indian Spectator.

Pair 2 is incorrectly matched: Sisir Ghosh

and Moti Lal Ghosh started Amrita Bazar

Patrika as a weekly first, it was first edited by Motilal Ghosh, who did not have a formal

university education. It had built its

readership as a rival to Bengalee which was

being looked after by Surendranath Banerjee.

Pair 3 is correctly matched: Devendra Nath Tagore started the Indian Mirror newspaper

in early 1862. This newspaper was published

in English.

Pair 4 is incorrectly matched: The Bengalee

an English newspaper was started by

Surendranath Banerjee and published from Calcutta from 1862 to 1931. It started as a

daily newspaper and subsequently turned

into a weekly. The Bengalee had a series of

renowned editors including Girish Chandra

Ghose and Surendranath banerjee.

71. Ans: (b)

Explanation:

Statement 1 is correct: Vernacular Press

Act was enacted in 1878 to curtail the

Page 20: ALL INDIA PRELIMS TEST SERIES - 2020 · the central government. was the court language. The Hindus for long Statement 2 is correct: It introduced Dyarchy in provinces, under which

20 AIPTS 2020 (HIS - 16) (E) Answer Key Byju’s Classes: 9873643487

freedom of the Indian-language (i.e., non-English) press. Proposed by Lord Lytton, then

viceroy of India (governed 1876–80), the act

was intended to prevent the vernacular press

from expressing criticism of British policies.

Statement 2 is incorrect: In vernacular press act magistrate action was final and no

appeal could be made in court of law.

Statement 3 is correct: Cranbrook, the

Secretary of State was not in favour of the

idea of pre-censorship clause and later on it

was done away with and a press commissioner was appointed to supply

authentic and accurate news to the press.The

entire Vernacular Press Act of 1878 was

repealed by Lord Ripon in 1882.

Statement 4 is correct: Surendranath Banerjee became the first Indian journalist to

be imprisoned under vernacular press act

1878.

72. Ans: (a) Explanation:

Pair 1 is incorrectly matched: The Calcutta

madrasah was established by warren

Hastings in 1781 for the study of Muslim law

and related subjects. Pair 2 is incorrectly matched: The Sanskrit

college was established by Jonathan Duncan,

the resident, at Benaras in 1791 for the study

of Hindu law and philosophy.

Pair 3 is correctly matched: Fort William

College was set up by Lord Richard Wellesley in 1800 for training of civil servants of the

company in languages and customs of

Indians.

73. Ans: (c) Explanation: In 1882, the government

appointed a commission under the

chairmanship of W.W. hunter to review the

progress of education in the country since

wood‘s dispatch of 1854. The hunter commission mostly contained its

recommendations to primary and secondary

education.

Statement 1 is correct: Commission drew

attention to inadequate facilities for female

education especially outside presidency towns and made recommendations for its spread.

Statement 2 is correct: It recommended

that secondary education should have two

divisions - literary education leading up to

university while vocational for commercial purposes.

Statement 3 is incorrect: Lord Rippon

(1880-1884) was the viceroy of British India

during this period.

74. Ans: (c) Explanation: SAMPARK: The aim is to trace

those who are Left to Follow Up and are to be

brought under Antiretroviral Therapy (ART)

services. In this direction, the ―Community Based Testing‖ to take HIV testing closer to

those in need, will help in fast-tracking the

identification of all who are HIV positive and

subsequently linking to the ART programme.

Counsellors are tasked to ensure tracking of

HIV positive patients through telephone and home visits in the case of those who drop out

between diagnosis. HIV & AIDS (Prevention

and Control) Act, 2017 was notified to

address the challenges related to the disease

in a comprehensive way.

75. Ans: (d)

Explanation: Gandhiji expressed his views

on education through a series of articles in

‗Harijan‘ in June 31, 1937, which later on developed into the Wardha Scheme of Basic

Education.

Statement 1 is correct: Wardha scheme had

provision of inclusion of basic handicraft in

syllabus.

Statement 2 is correct: First seven years of schooling was to be an integral part of a free

and compulsory nationwide education

system.

Statement 3 is correct: Gandhiji asserted

that no education is possible through foreign medium and all elementary education must

be imparted through the mother tongue. His

views reflected in the scheme

Statement 4 is correct: Wardha Education

Conference appointed a committee under the

Chairmanship of Dr. Zakir Hussain to prepare a detailed education plan and

syllabus on the lines of the resolutions

passed in the conference.

76. Ans: (c) Explanation: The British Colonial

government had committed India into the

Second World War without the consent of the

Indian people. To oppose this decision by the

foreign government, the Congress party decided to launch individual satyagraha.

Underlying this decision there was a strategy

of preparing their supporters and the party

organisation for the mass movement which

was to follow. Statement 1 is incorrect: First Satyagrahi

was Vinoba Bhave, while Nehru was the

second one.

Statement 2 is incorrect: 'Chalo Delhi'

slogan was given by Bose in 1944 during

INA's March towards India. During the

Page 21: ALL INDIA PRELIMS TEST SERIES - 2020 · the central government. was the court language. The Hindus for long Statement 2 is correct: It introduced Dyarchy in provinces, under which

21 AIPTS 2020 (HIS - 16) (E) Answer Key Byju’s Classes: 9873643487

Individual Satyagraha, the satyagrahi would give a speech at a place and move to another

place while trekking towards delhi. Thus, this

movement toward the capital was popularly

called Delhi Chalo.

Statement 3 is correct: August offer was given to pacify INC as the Congress party

opposed the British government‘s decision to

include India into the Second World War

without the consent of the Indian people. INC

rejected August offer at its meeting at Wardha

in August 1940, demanding complete freedom from colonial rule. After this, Mahatma

Gandhi initiated the Individual Satyagraha to

affirm the right to free speech.

77. Ans: (d) Explanation: Din Bandhu Mitra‘s play, Neel

Darpan, gained great fame for vividly

portraying the oppression by the planters.

The Indigo Revolt (1858) or Nilbidroha in

Bengali was the revolt of the indigo farmers against the indigo planters. It was just one

year after the Sepoy Revolt Bengal saw one

more important revolt in its history. The evils

of Kulin Brahminism, widow marriage

prohibition, quackery, fanaticism, have been depicted by it with great effect in the play.

78. Ans: (b)

Explanation:

Statement 1 is correct: Swadeshi movement was the first mass movement launched in the

struggle for independence. People came out in

large numbers and bathed in River Ganga.

Rabindranath Tagore composed a national

song for the occasion which was sung by the

masses in the streets. They boycotted the British goods in their procession and soon

the movement gained momentum.

Statement 2 is incorrect: The congress in

its 1906 session at Benaras passed a

resolution for the swadeshi and self governance or swaraj. The swadeshi

movement was launched in 1905 itself,

official congress resolution came the next

year.

Statement 3 is correct: This movement

sprung up as a reaction to the step taken by the British government to partition Bengal.

People came out in large numbers to show

their descent and picketed shops selling

foreign garments. There were organized

bonfires in various parts where people burnt foreign products.

79. Ans: (a) Explanation: Sustainable Alternative

Towards Affordable Transportation (SATAT)

initiative to promote Compressed Biogas as

an alternative, green transport fuel. The initiative is aimed at providing a Sustainable

Alternative Towards Affordable

Transportation as a developmental effort that

would benefit both vehicle-users as well as

farmers and entrepreneurs. This initiative

holds great promise for efficient municipal solid waste management and in tackling the

problem of polluted urban air due to farm

stubble-burning and carbon emissions. Use

of CBG will also help bring down dependency

on crude oil imports and in realising the Prime Minister‘s vision of enhancing farmers‘

income, rural employment and

entrepreneurship.

80. Ans: (a) Explanation:

Statement 1 is correct: In 1884, Bal

Gangadhar Tilak, founded the Deccan

Education Society. The following year,

Gokhale joined one of the society‘s schools as

a teacher. In 1889, Ranade appointed him editor of the quarterly journal of the Poona

Sarvajanik Sabha.

Statement 2 is correct: In 1905 Gokhale

founded the Servants of India Society, whose

members dedicated themselves to serving the nation-in-the-making. The founding of this

brotherhood was the outcome of Gokhale's

convictions that if the masses were to be

liberated to function as active members of

free India, they must have a band of selfless

and intelligent workers who would dedicate their lives to the service of the nation.

Statement 3 is incorrect: ‗Servants of India‘

were required to ‗work for the advancement

of all Indians, regardless of caste or creed‘.

The workers were required to take the vow of renunciation, give up all ideas of selfishness,

pride, fame and be one with their work and

duty.

81. Ans: (b) Explanation: George Yule became the first

British president of INC and presided over the

Allahabad session in 1888. He was a Scottish

merchant in England and served as Sheriff of

Calcutta and as President of the Indian Chamber of Commerce.

Option (a) is incorrect: Sir William

Wedderburn was a Scottish civil servant and

politician who was a Liberal Party member of

Parliament (MP). Wedderburn was one of the

founding members of the Indian National

Page 22: ALL INDIA PRELIMS TEST SERIES - 2020 · the central government. was the court language. The Hindus for long Statement 2 is correct: It introduced Dyarchy in provinces, under which

22 AIPTS 2020 (HIS - 16) (E) Answer Key Byju’s Classes: 9873643487

Congress. He was also the president of Congress in 1889.

Option (c) is incorrect: Alfred John Webb

was an Irish Quaker from a family of activist

printers. He became an Irish Parliamentary

Party politician and Member of Parliament (MP), as well as a participant in nationalist

movements around the world. He supported

Butt's Home Government Association and the

United Irish League. At Madras in 1894, he

became the third non-Indian (after George

Yule and William Wedderburn) to preside over the Indian National Congress.

Option (d) is incorrect: Henry Cotton had a

long career in the Indian Civil Service, during

which he was sympathetic to Indian

nationalism. In 1904, he served as President of the Indian National Congress.

82. Ans: (b)

Explanation:

Statement 1 is incorrect : RISAT- 2BR1 is a radar imaging earth observation satellite

which uses the Synthetic Aperture Radar

system which allows the mapping of ground

surface. The satellite sends microwave pulses

to Earth. The pulses return back to the satellite and the sensor makes a picture out

of the returned echoes. This enables the

satellite to provide services in the field of

agriculture, forestry, disaster management as

well as for reconnaissance purposes. The

launch marked the 50th flight of the PSLV mission on board the PSLV C48.

Statement 2 is correct: RISAT 2BR1 is

considered as an all weather, day night

satellite which does not depend on visual

form to map an area. The reflection of pulses from the ground surface enables the satellite

to perform its task even in pitch dark or

cloudy conditions. The continuous detection

of pulses allows the satellite to have an

accurate mapping of an area and detect any

minor movements. This ability also makes the satellite suitable for maritime surveillance.

83. Ans: (b)

Explanation:

Statement 1 is incorrect: National Herald was launched in 1938 as a daily newspaper

by Pandit Jawarharlal Nehru. The newspaper

shared the vision of its founder and the best

values of the Freedom Movement – that of

building a modern, democratic, just, equitable, liberal and socially harmonious

nation, free of sectarian strife. It was banned

by British government in 1942 during the

Quit India movement. It was one of the major

English language newspapers in India after

the end of the British Raj. The newspaper

ceased operations in 2008 for financial reasons. In 2016, it was relaunched as a

digital publication.

Statement 2 is correct: The Indian

Sociologist was an Indian nationalist journal

in the early 20th century, started by Shyamji Krishnavarma in London. One of the reasons

to publish it abroad was to remain immune

from Press Acts. It also made an attempt to

disseminate patriotism and nationalism

among the youth so that they become active

participants in the fight against Britishers as part of the revolutionary activities.

84. Ans: (d)

Explanation: Statement 1 is incorrect: The British

government announced the cancellation of

the partition of Bengal in 1911 whereas

World War was started in 1914. Hence,

annulment of the partition of Bengal

happened much before WWI. Statement 2 is correct: Two Home Rule

Leagues were started in 1915-16, one under

the leadership of Lokmanya Tilak and other

under the leadership of Annie Besant, and S.

Subramaniya Iyer. It occurred during the period of World War I which is from 1914 to

1918. Statement 3 is correct: The Ghadar

Party pledged to wage revolutionary war

against the British in India. As soon as the

First World War broke out in 1914, the

Ghadarites decided to send arms and men to India to start an uprising with the help of

soldiers and local revolutionaries. Finally, 21

February 1915 was fixed as the date for an

armed revolt in the Punjab. Unfortunately,

the authorities came to know and took immediate action. The rebellious regiments

were disbanded and their leaders were either

imprisoned or hanged.

Statement 4 is correct: In 1917, Gandhiji

led a successful satyagraha campaign fighting

for the rights of the indigo planters at Champaran. Champaran was a turning point

in India's freedom struggle. For the first time

with peaceful means, Gandhiji declared that

the British could not order me about in my

own country'. Simultaneously, he tapped the power of the present masses and awoke their

spirit of dignity and self-reliance. The World

war started in 1914 and lasted till November

1918.

Statement 5 is incorrect: The British

government announced the cancellation of the partition of Bengal in 1911. At the same

time the seat of the Central Government was

shifted to Delhi from Calcutta, and World War

I started three years later in 1914.

Page 23: ALL INDIA PRELIMS TEST SERIES - 2020 · the central government. was the court language. The Hindus for long Statement 2 is correct: It introduced Dyarchy in provinces, under which

23 AIPTS 2020 (HIS - 16) (E) Answer Key Byju’s Classes: 9873643487

85. Ans: (a) Explanation: While agreeing to hear petitions

on alleged police excesses on students in

Jamia Millia Islamia and Aligarh Muslim

University, a Supreme Court Bench headed by Chief Justice of India S A Bobde on

Monday expressed displeasure over rioting

and destruction of public property. Despite a

law against the destruction of property,

incidents of rioting, vandalism, and arson

have been common during protests across the country. In 2009, in the Destruction of

Public & Private Properties v State of AP and

Others, the Supreme Court issued guidelines

based on the recommendations of the

Thomas Committee.

86. Ans: (c)

Explanation: When the country was

preparing for the start of the civil

disobedience movement in 1930, the students of Assam played a key role. To discourage

and stop students to participate in CDM J.R.

Cunningham, the then powerful director of

public information of Assam, issued the

cunningham circular in 1930 imposing

blanket ban on any anti-British and pro-swadeshi activity by students. It forced the

parents, guardians, and students to furnish

assurance of good behaviour and also asked

them to sign an undertaking that they would

have to quit their schools and colleges if they participated in anti-government

demonstrations or movements. In Assam, a

powerful agitation led by students was

launched against the Cunningham circular.

87. Ans: (a) Explanation: The Irwin Declaration or also

known as Deepavali Declaration (announced

on the day of Deepavali festival) was a

statement made by Lord Irwin, then Viceroy of India, on 31 October 1929 regarding the

status of India in the British empire. It was

intended to placate leaders of the Indian

nationalist movement who had become

increasingly vocal in demanding dominion

status for India. The Declaration was a five-line statement in simple non-legal language.

It attempted to clarify to its British and

Indian audiences that the intention of the

British government was to facilitate India to

attain dominion status in the future. However, there was no mention of any

timeline. In India, nationalist leaders

welcomed the Declaration and radically

changed their mode of engagement with the

British government: they now wanted all

negotiations between Indian political leaders

and Britain to be about the formalisation of dominion status for India and the framing of

a new Constitution.

88. Ans: (d) Explanation: In December 1928, during all

parties' meetings Jinnah proposed some

amendments on the Nehru‘s Report. He

proposed ‗Fourteen Points‘ for safeguarding

the rights and interests of the Muslims in any

future constitution of the country. Jinnah‘s Fourteen Points included:

1. Federal constitution with residual powers

with the provinces.

2. Provincial autonomy.

3. No constitutional amendment without the agreement of the states.

4. All legislatures and elected bodies to have

adequate Muslim representation without

reducing Muslim majority in a province to

minority or equality.

5. Adequate Muslim representation of Muslims in the services and in self-

governing bodies.

6. 1/3rd representation of Muslims in the

Central Legislature.

7. 1/3rd Muslim members in the central and state cabinets.

8. Separate electorates.

9. No bill to be passed in any legislature if

3/4th of a minority community considers

it against its interests.

10. Any reorganisation of territories not to affect the Muslim majority in Bengal,

Punjab and the NWFP.

11. Separation of Sindh from Bombay

Presidency.

12. Constitutional reforms in the NWFP and Balochistan.

13. Full religion freedom for all communities.

14. Protection of the religious, cultural,

educational and language rights of

Muslims.

89. Ans: (c)

Explanation:

Statement 1 is correct: During the peaceful

strike led by Gandhi, he underwent a hunger

strike for the first time in India for the cause of workers.

Statement 2 is correct: Gandhi used

Satyagraha and hunger strike for the first

time during an industrial dispute between the

owners and workers of a cotton mill in Ahmedabad. The owners wanted to withdraw

the plague bonus to the workers while the

workers were demanding a hike of 35% in

their wages. The strike was successful and

the workers were granted the wage hike they

wanted.

Page 24: ALL INDIA PRELIMS TEST SERIES - 2020 · the central government. was the court language. The Hindus for long Statement 2 is correct: It introduced Dyarchy in provinces, under which

24 AIPTS 2020 (HIS - 16) (E) Answer Key Byju’s Classes: 9873643487

90. Ans: (c) Explanation:

Statement 1 is correct: Bhagat Singh

actively participated in the Non-Cooperation

movement. He later distanced himself from Gandhi‘s non-violent approach after the

Chauri Chaura incident in 1922 and aligned

himself with the Young Revolutionary

Movement.

Statement 2 is incorrect: Sachindranath

Sanyal wrote Bandi Jeevan (A Life of Captivity, 1922) during his internment at

Cellular Jail. Bhagat Singh writings are

compiled as ―Jail Diary and Other Writings ―

Statement 3 is incorrect: The famous

slogan Inquilab Zindabad was given by Hasrat Mohani. This famous slogan inspired

the activities of the Hindustan Socialist

Republican Association particularly

Ashfaqulla Khan, Bhagat Singh and

Chandrasekhar.

91. Ans: (b)

Explanation: ―Power of Siberia‖ is a massive

gas pipeline linking one of the most remote

parts of Russia with a far-flung region

of China. The pipeline stretches more than 3,000km (1,864 miles). Under this

pipeline project, Russia will deliver 1 trillion

cubic meters of natural gas to China over

the next 30 years. The pipeline will pass

through the deltas of the Yangtze and Amur rivers of China. The new gas pipeline is the

largest gas infrastructure in the entire

Russian Far East connecting the enormous

Siberian fields of Kovyktinskoye and

Chayandinskoye to Blagovehensk, the

Russian city on the Amur river that marks the border between the Russian Federation

and the People‘s Republic of China.

92. Ans: (c) Explanation: The Cripps Mission was sent by

the British government to India in March

1942. It was headed by Sir Richard Stafford

Cripps, a labour minister in Winston

Churchill‘s coalition government in Britain.

Statement 1 is correct: Japan was advancing outside the eastern borders of

India and the fall of Burma was a jolt to the

British in the Second World War. The threat

of a Japanese invasion on India was looming

and Indian support was essential for Britain‘s war efforts. Thus, the Cripps Mission was

sent by the British government to India to

obtain Indian cooperation for the British war

efforts in the 2nd World War.

Statement 2 is correct: One of the

significance of Cripps Mission was that for

the first time, the British government acknowledged India‘s right to be a dominion

and proposed that Indians could frame their

own Constitution.

93. Ans: (a) Explanation:

Statement 1 is correct: The Komagata Maru

incident is about a Japanese steamship

called ‗Komagata Maru‘ that voyaged from

Hong Kong (part of British Empire) to Vancouver, British Columbia in Canada

passing through Shanghai, China to

Yokohama (Japan) in the year 1914, carrying

376 passengers from Punjab, part of British

India. The ‗Komagata Maru‘ incident was quoted largely at that time by various Indian

groups to underscore inconsistencies in

Canadian immigration laws. The emotions

ignited after this incident were taken

advantage of by the Indian revolutionaries,

particularly, the Ghadar Party members to gather support from the people for their

cause.

Statement 2 is incorrect: The party was

headquartered in San Francisco, United

States. Original name of the Ghadar Party was Pacific Coast Hindustan Association.

Statement 3 is correct: Sohan Singh

Bhakna and Lala Hardayal were the founding

members of the Ghadar Party. Key members

included Bhai Parmanand, Sohan Singh

Bhakna, Bhagwan Singh Gyanee, Har Dayal, Tarak Nath Das, Kartar Singh Sarabha,

Abdul Hafiz Mohamed Barakatullah,

Rashbehari Bose, and Gulab Kaur.

94. Ans: (c) Explanation:

Option (a) is correct: The Tebhaga

movement saw involvement of girls at a high

level. It saw widespread independent action of

the landless and poor peasant women:, belonging to dalit and tribal communities.

Through their own initiative they formed Nari

Bahinis or women's brigades and resisted the

colonial police with whatever weapon they

could lay their hands on.

Option (b) is correct: The Government of India Act of 1935 extended the principle of

communal representation by providing

separate electorates for depressed classes

(scheduled castes), women and labour

(workers). Option (c) is incorrect: Women's Indian

Association was started by enlightened

European and Indian ladies, the most

important being Margaret Cousins and Annie

Besant.

Page 25: ALL INDIA PRELIMS TEST SERIES - 2020 · the central government. was the court language. The Hindus for long Statement 2 is correct: It introduced Dyarchy in provinces, under which

25 AIPTS 2020 (HIS - 16) (E) Answer Key Byju’s Classes: 9873643487

Option (d) is correct: Aruna Asaf Ali popularly known as the 'Grand Old Lady' of

the Independence Movement, was one of the

independence activists. She is known for

hoisting the Indian flag at the Gowalia Tank

Maidan in Mumbai during the Quit India Movement. In 1932, she went on a hunger

strike in Tihar Jail against the mistreatment

of the political prisoners, which led to an

improvement in their living conditions.

95. Ans: (c) Explanation: Armed with its new majority,

the Britain government has published a

revised version of its Withdrawal Agreement

Bill (WAB) to ensure that it can take the UK out of the EU. It sets out exactly how the UK

will make "divorce bill" payments to the EU

for years to come. It repeals the European

Communities Act, which took the UK into the

EU, but then reinstates it immediately until

the end of 2020 when the transition period ends. It sets out areas in which the European

Court of Justice still plays a role in the UK.

96. Ans: (c)

Explanation: Statement 1 is correct: In 1907, Surat

Session, Surat Split happened. The

differences between the moderates and the

extremists became official in the Surat

session of the Indian National Congress (INC) in 1907. The extremists wanted Lala Lajpat

Rai or Bal Gangadhar Tilak to be the

President. But the moderates wanted Rash

Behari Ghosh as President. Ultimately, Rash

Behari Ghosh became the president in the

session which was held at Surat. Both sides differed in their demand and were firm on

that and neither was willing to find a common

path. Thus, a split occurred between them.

Statement 2 and 3 are incorrect: It was the

Calcutta session of 1906, where Dadabhai Naoroji in his presidential address declared

that the goal of Indian national movement

was ‗self-government‘ or swaraj, like that of

the United Kingdom or other colonies.

97. Ans: (b) Explanation:

Option (a) is incorrect: Tilak played an

important role in the independence

movement of India and also launched All India Home Rule League in 1916 along with

Annie Besant. But he didn‘t set up Central

Hindu College (CHC) at Benares.

Option (b) is correct: On 1 October 1847,

Annie Besant was born in London. She came

to India for the first time in 1893 as part of

the Theosophical Society. Annie Besant was the society‘s president from 1907 to 1933.

Besant played an important role in the

independence movement of India. Besant set

up the Central Hindu College (CHC) at

Benares. She met Madan Mohan Malaviya who joined forces to found the Banaras

Hindu University in 1916. The CHC became

the university‘s first constituent college.

Besant launched the All India Home Rule

League in 1916 along with Bal Gangadhar

Tilak. The League was the first Indian political party which advocated self-rule as its

motto. And in contrast to the INC which met

once a year, the League worked throughout

the year.

Option (c) is incorrect: Prabhu Narayan Singh was the Maharaja of the princely state

of Benares, (Royal House of Benares),

currently known as Varanasi or Benaras,

during the British rule in India. He was not

associated with the home rule league

movement. Option (d) is incorrect: The massacre of

hundreds of Indians by the British at

Amritsar in 1919 prompted Motilal to join

Mahatma Gandhi‘s noncooperation

movement, giving up his career in law and changing to a simpler, non-Anglicized style of

life. In 1921 both he and Jawaharlal were

arrested by the British and jailed for six

months. He played no role in establishing

CHC.

98. Ans: (b)

Explanation:

Statement 1 is incorrect: The printing press

was brought to India by the Portugese. The art of printing first entered India through

Goa. In a letter to St. Ignatius of Loyola,

dated 30 April 1556, Father Gasper Caleza

speaks of a ship carrying a printing press

setting sail for Abyssinia from Portugal, with

the purpose of helping missionary work in Abyssinia.

Statement 2 is correct: Hicky‘s Bengal

Gazette was the first English-language

newspaper published in India. It was founded

in Calcutta, capital of British India at the time, by Irishman James Augustus Hicky in

1779.

99. Ans: (c)

Explanation: United Independent Bengal Movement or Bose-Suhrawardy Plan was a

proposal to solve the communal question on

the eve of the termination of British rule in

India. In April-May 1947 it became clear that

the Partition of India was a real possibility.

Huseyn shaheed suhrawardy, the Premier of

Page 26: ALL INDIA PRELIMS TEST SERIES - 2020 · the central government. was the court language. The Hindus for long Statement 2 is correct: It introduced Dyarchy in provinces, under which

26 AIPTS 2020 (HIS - 16) (E) Answer Key Byju’s Classes: 9873643487

the province of Bengal, formally launched his idea of a sovereign state for undivided Bengal.

Almost simultaneously sarat chandra bose

came forward with his proposal for a

Sovereign Socialist Republic of Bengal. There

had been differences of opinion between Suhrawardy and Sarat Bose regarding the

sovereign status of Bengal, but the primary

motive of both of them was to prevent the

partition of the province. Thus, it is

associated with the united Bengal as a third

successor state to British India after India and Pakistan.

100. Ans: (c)

Explanation: Statement 1 is incorrect: A trade

mechanism ‗Instrument in Support of Trade

Exchanges (INSTEX)‘ was established

by France, Germany and the United

Kingdom in January 2019 to allow European entities to maintain trade with Iran. Six

Countries - Belgium, Denmark, Finland, the

Netherlands, Norway and Sweden have

recently joined INSTEX. It was not devised by

the European Union. Statement 2 is correct: The mechanism has

been designed to circumvent U.S. sanctions

against trade with Iran by avoiding the use of

the dollar. This Paris-based

mechanism functions as a clearing

house allowing Iran to continue to sell oil and import other products or services in

exchange.

Statement 3 is correct: The mechanism is

devised on the basis of barter system. In the

barter system, participants in a transaction directly exchange goods or services for other

goods or services without using a medium of

exchange, such as money.